You are on page 1of 91

LICENSURE EXAMINATION FOR TEACHERS (LET)

Refresher Course

Content Area: MATHEMATICS


Focus: ANALYTIC GEOMETRY
Prepared by: Daisy de Borja-Marcelino
Competencies:
1. Solve problems involving coordinates of a point, midpoint of a line segment, and distance between two
points.
2. Determine the equation of the line relative to given conditions: slope of a line given its graph, or its
equation, or any two points on it.
3. Determine the equation of a non-vertical line given a point on it and the slope of a line, which is either
parallel or perpendicular to it.
4. Solve problems involving
a. the midpoint of a line segment, distance between two points, slopes of lines, distance
between a point and a line, and segment division.
b. a circle, parabola, ellipse, and hyperbola.
5. Determine the equations and graphs of a circle, parabola, ellipse and hyperbola.

I. The Cartesian Plane

Below is a diagram of a Cartesian plane or a rectangular coordinate system, or a coordinate plane.

The two axes separate the plane into four


regions called quadrants. Points can lie in one of
the four quadrants or on an axis. The points on
the x-axis to the right of the origin correspond to
positive numbers; while to the left of the origin
correspond to negative numbers. The points on
the y-axis above the origin correspond to positive
numbers; while below the origin correspond to
negative numbers.
An ordered pair of real numbers, called the coordinates of a point, locates a point in the Cartesian plane.
Each ordered pair corresponds to exactly one point in the Cartesian plane.

The following are the points in the figure on the right:


A(-6,3), B(-2,-3), C (4,-2), D(3,4), E(0,5), F(-3,0).

For numbers 1-2, use the following condition: Two insects M and T are initially at a point A(-4, -7) on a
Cartesian plane.

1. If M traveled 7 units to the right and 8 units downward, at what point is it now?
Solution: (-4+7, -7-8) or (-3,-15)
2. If T traveled 5 units to the left and 11 units downward, at what point is it now?
Solution: (-4-5, -7-11) or (-9, -18)

II. The Straight Line

A. Distance Between Two Points


A. The distance between two points (x1,y1) and (x2,y2) is given by ( x1  x2 ) 2  ( y1  y2 ) 2 .
Example: Given the points A(2,1) and B(5,4). Determine the length AB.

Solution: AB = 2  52  1  42   92   32  9  9  18 or 9  2 or 3 2 .


Exercises: For 1-2, use the following condition: Two insects L and O are initially at a point (-1,3) on a Cartesian plane.
1. If L traveled 5 units to the left and 4 units upward, at what point is it now?
A) (-6, 7) B) (4, 7) C) (-6, -1) D) (4, -7)

2. If O traveled 6 units to the right and 2 units upward, at what point is it now?
A) (7, 5) B) (5,5) C) (-7, 5) D) (-5, -5)

3. Two buses leave the same station at 9:00 p.m. One bus travels at the rate of 30 kph and the other travels at 40
kph. If they go on the same direction, how many km apart are the buses at 10:00 p.m.?
A) 70 km B) 10 km C) 140 km D) 50 km

4. Two buses leave the same station at 8:00 a.m. One bus travels at the rate of 30 kph and the other travels at 40
kph. If they go on opposite direction, how many km apart are the buses at 9:00 a.m.?
A) 70 km B) 10 km C) 140 km D) 50 km

5. Two buses leave the same station at 7:00 a.m. One bus travels north at the rate of 30 kph and the other travels
east at 40 kph. How many km apart are the buses at 8:00 a.m.?
A) 70 km B) 10 km C) 140 km D) 50 km

6. Which of the following is true about the quadrilateral with vertices A(0,0), B(-2,1), C(3,4) and D(5,3)?
i) AD and BC are equal
ii) BD and AC are equal
iii) AB and CD are equal

A) both i and iii B) ii only C) both ii and iii D) i, ii, and iii

7. What is the distance between (-5,-8) and (10,0)?


A) 17 B) 13 C) 23 D) -0.5

B. Slope of a line
y1  y2 y2  y1
a) The slope of the non-vertical line containing A(x1,y1) and B(x2,y2) is m or m  .
x1  x2 x2  x1
b) The slope of the line parallel to the x-axis is 0.
c) The slope of the line parallel to the y-axis is undefined.
d) The slope of the line that leans to the right is positive.
e) The slope of the line that leans to the left is negative.

C. The Equation of the line


In general, a line has an equation of the form ax + by + c = 0 where a, b, c are real numbers and that a
and b are not both zero.

D. Different forms of the Equation of the line


 General form: ax + by + c = 0.
 Slope-intercept form: y = mx + b, where m is the slope and b is the y-intercept.
 Point slope form: y  y1  m( x  x1 ) where (x1, y1) is any point on the line.
y2  y1
 Two point form: y  y1  ( x  x1 ) where (x1, y1) and (x2, y2) are any two points on the line.
x2  x1
x y
 Intercept form:   1 where a is the x-intercept and b is the y-intercept.
a b

Reminders:
 A line that leans to the right has positive slope. The steeper the line, the higher the slope is.

p q r
The slopes of lines p, q, r are all positive. Of the three slopes, the slope of line p is the lowest, the slope of
r is the highest.

 A line that leans to the left has negative slope. The steeper the line, the lower the slope is.

t s u

The slopes of lines t, s, u are all negative. Of the three slopes, t is the highest, while u has the lowest (because the
values are negative.)

Exercises
1. What is the slope of 5x - 4y + 12 = 0 ?
A)1.25 B) -1.25 C) 0.8 D) -0.8

2. What is the slope of x = -9?


A) 4 B) 1 C) 0 D) undefined

3. What is the slope of y= 12?


A) 7 B) 1 C) 0 D) undefined

x y
4. What is the slope of + = 1?
4 9
A) 0.4 B) 2.25 C) - 0.4 D) - 2.25

E. Parallel and Perpendicular lines


Given two non-vertical lines p and q so that p has slope m1 and q has slope m2.
 If p and q are parallel, then m1 = m2.
 If p and q are perpendicular to each other, then m1m2 = -1.

F. Segment division
Given segment AB with A(x1,y1) and B(x2,y2).
x1  x2 y1  y2
 The midpoint M of segment AB is M ( , ).
2 2
r AP r1
 If a point P divides AB in the ratio 1 so that  , then the coordinates of P(x,y) can be obtained
r2 PB r2
r x  r2 x1 r y  r2 y1
using the formula x  1 2 and y  1 2 .
r1  r2 r1  r2
G. Distance of a point from a line

The distance of a point A(x1,y1) from the line Ax + By + C = 0 is given by


Ax1  By1  C
d .
A2  B 2
Exercises

1. Write an equation in standard form for the line passing through (–2,3) and (3,4).
a. 5x – y = -13 b. x – 5y = 19 c. x – y = -5 d. x – 5y = –17

2. Write an equation in slope intercept form for the line with a slope of 3 and a y-intercept of 28.
a. y = –3x + 28 b. y = 0.5x + 28 c. y = 3x + 28 d. y = 3x + 21

3. Write the equation in standard form for a line with slope of 3 and a y-intercept of 7.
a. 3x – y = –7 b. 3x + y = 7 c. 3x + y = 7 d. –3x + y = –7

4. Which of the following best describes the graphs of 2x – 3y = 9 and 6x – 9y = 18?


a. Parallel b. Perpendicular c. Coinciding d. Intersecting

5. Write the standard equation of the line parallel to the graph of x – 2y – 6 = 0 and passing through (0,1).
a. x + 2y = –2 b. 2x – y = –2 c. x – 2y = –2 d. 2x + y = –2
6. Write the equation of the line perpendicular to the graph of x = 3 and passing through (4, –1).
a. x – 4 = 0 b. y + 1 = 0 c. x + 1 = 0 d. y – 4 = 0
7. For what value of d will the graph of 6x + dy = 6 be perpendicular to the graph 2x – 6y = 12?
a. 0.5 b. 2 c. 4 d. 5

III. Conic Section


A conic section or simply conic, is defined as the graph of a second-degree equation in x and y.

In terms of locus of points, a conic is defined as the path of a point, which moves so that its distance from
a fixed point is in constant ratio to its distance from a fixed line. The fixed point is called the focus of the conic, the
fixed line is called the directrix of the conic, and the constant ratio is called the eccentricity, usually denoted by e.

If e < 1, the conic is an ellipse. (Note that a circle has e=0.)


If e = 1, the conic is a parabola.
If e > 1, the conic is hyperbola.

A. The Circle

1. A circle is the set of all points on a plane that are equidistant from a fixed point on the plane. The fixed point
is called the center, and the distance from the center to any point of the circle is called the radius.

2. Equation of a circle

a) general form: x2 + y2 + Dx + Ey + F = 0
b) center-radius form: (x – h)2 + (y – k)2 = r2 where the center is at (h,k) and the radius is equal to r.

3. Line tangent to a circle


A line tangent to a circle touches the circle at exactly one point called the point of tangency. The tangent
line is perpendicular to the radius of the circle, at the point of tangency.

Exercises
y
For items 1-2, use the illustration on the right. 2

1.5

1. Which of the following does NOT lie on the circle?


1
a. (3,-1) b. (3,0)
0.5
c. (2,-1) d. (3,-2)
0 x
-3.5 -3 -2.5 -2 -1.5 -1 -0.5 0 0.5 1 1.5 2 2.5 3 3.5

2. What is the equation of the graph? -0.5

a. y  x  3  1
2
b. ( y  1)  x  3  1
2
-1

c. ( y  1)  x  3  1
2
d. ( y  1)  x  3  1
2 -1.5

-2

B. The Parabola

1. Definition. A parabola is the set of all points on a plane that are equidistant from a
fixed point and a fixed line of the plane. The fixed point is called the focus and the fixed line is the directrix.

2. Equation and Graph of a Parabola

a) The equation of a parabola with vertex at the origin and focus at (a,0) is y2 = 4ax. The parabola
opens to the right if a > 0 and opens to the left if a < 0.

b) The equation of a parabola with vertex at the origin and focus at (0,a) is x2 = 4ay. The parabola
opens upward if a > 0 and opens downward if a < 0.

c) The equation of a parabola with vertex at (h , k) and focus at (h + a, k) is (y – k)2 = 4a(x – h).
The parabola opens to the right if a > 0 and opens to the left if a < 0.

d) The equation of a parabola with vertex at (h , k) and focus at (h, k + a) is (x – h)2 = 4a(y – k).

e) The parabola opens upward if a > 0 and opens downward if a < 0.

f) Standard form: (y – k)2 = 4a(x – h) or (x – h)2 = 4a(y – k)

g) General form: y2 + Dx + Ey + F = 0, or x2 + Dx + Ey + F = 0
3. Parts of a Parabola

a) The vertex is the point, midway between the focus and the directrix.

b) The axis of the parabola is the line containing the focus and perpendicular to the directrix. The parabola
is symmetric with respect to its axis.

c) The latus rectum is the chord drawn through the focus and parallel to the directrix (and therefore
perpendicular to the axis) of the parabola.

d) In the parabola y2=4ax, the length of latus rectum is 4a, and the endpoints of the latus rectum are (a, -
2a) and (a, 2a).

In the figure at the right, the vertex of the parabola is the origin,
the focus is F(a,o), the directrix is the line containing LL ' ,
the axis is the x-axis, the latus rectum is the line containing CC' .

16
The graph of x  
2
y. The graph of (y-2)2 = 8 (x-3).
3

C. Ellipse

1. An ellipse is the set of all points P on a plane such that the sum of the distances of P from two fixed points F’
and F on the plane is constant. Each fixed point is called focus (plural: foci).

2. Equation of an Ellipse

a) If the center is at the origin, the vertices are at ( a, 0), the foci are at ( c,0), the endpoints of the minor
x2 y2
axis are at (0,  b) and b  a  c , then the equation is
2 2 2
  1.
a 2 b2
b) If the center is at the origin, the vertices are at (0,  a), the foci are at (0,  c), the endpoints of the
x2 y2
minor axis are at ( b, 0) and b  a  c , then the equation is
2 2 2
 1.
b2 a 2
c) If the center is at (h, k), the distance between the vertices is 2a, the principal axis is horizontal and
( x  h) 2 ( y  k ) 2
b 2  a 2  c 2 , then the equation is   1.
a2 b2
d) If the center is at (h, k), the distance between the vertices is 2a, the principal axis is vertical and
( y  k ) 2 ( x  h) 2
b 2  a 2  c 2 , then the equation is   1.
a2 b2

4. Parts of an Ellipse 2 B(0,b 2


(c, ba ) (c, ba )
For the terms described below, refer to the ellipse )
shown with center at O, vertices at V’(-a,0) and V(a,0),
foci at F’(-c,0) and F(c,0), endpoints of the minor axis
V’(- F’(- O F(c,0 V(a,0 x
at B’(0,-b) and B(0,b), endpoints of one latus rectum
a,0) c,0) ) b2 )
b2 b2 b2
(  c,  a ) ( c,  a )
at G’ (-c, 
) and G(-c, ) and the other at B’(0,-
a a
b)
b2 b2
H’ (c,  ) and G(c, ).
a a

a) The center of an ellipse is the midpoint of the segment joining the two foci. It is the intersection of the
axes of the ellipse. In the figure above, point O is the center.
b) The principal axis of the ellipse is the line containing the foci and intersecting the ellipse at its vertices.
The major axis is a segment of the principal axis whose endpoints are the vertices of the ellipse. In the
figure, V 'V is the major axis and has length of 2a units.
c) The minor axis is the perpendicular bisector of the major axis and whose endpoints are both on the
ellipse. In the figure, B' B is the minor axis and has length 2b units.
d) The latus rectum is the chord through a focus and perpendicular to the major axis. G' G and H ' H
2b 2
are the latus rectum, each with a length of .
a
y y

(2,6)
(0, 3) (8,5)
(-6,4)
(-4, 9 ) (4, 9 )
5 5
(2,1)
(-5,0) (-4,0) (4,0) (5,0) (-8,1) (12,1)
O x O x
(4,- 9 )
(8,3)
(-4,- 9 ) 5
5
(0, -3) (2,-4)
x2 y2 ( x  2) 2 ( y  1) 2
The graph of   1. The graph of   1.
25 9 100 25

4. Kinds of Ellipses
a) Horizontal ellipse. An ellipse is horizontal if its principal axis is horizontal. The graphs above are both
horizontal ellipses.
b) Vertical ellipse. An ellipse is vertical if its principal axis is vertical.

D. The Hyperbola

1. A hyperbola is the set of points on a plane such that the difference of the distances of each point on the set
from two fixed points on the plane is constant. Each of the fixed points is called focus.
2. Equation of a hyperbola
a) If the center is at the origin, the vertices are at ( a, 0), the foci are at ( c,0), the endpoints of the minor
x2 y2
axis are at (0,  b) and b  c  a , then the equation is
2 2 2
 1.
a 2 b2
b) If the center is at the origin, the vertices are at (0,  a), the foci are at (0,  c), the endpoints of the minor
y2 x2
axis are at ( b, 0) and b  c  a , then the equation is 2  2  1 .
2 2 2

a b
c) If the center is at (h, k), the distance between the vertices is 2a, the principal axis is horizontal and
( x  h) 2 ( y  k ) 2
b 2  c 2  a 2 , then the equation is   1.
a2 b2
d) If the center is at (h, k), the distance between the vertices is 2a, the principal axis is vertical and
( y  k ) 2 ( x  h) 2
b  c  a , then the equation is
2 2 2
 1
a2 b2

2. Parts of a hyperbola

For the terms described below, refer to the hyperbola shown which has its center at O, vertices at V’(-a,0)
b2 b2
and V(a,0), foci at F’(-c,0) and F(c,0) and endpoints of one latus rectum at G’ (-c,  ) and G(-c, ) and the
a a
b2 b2
other at H’ (c,  ) and H(c, ).
a a

a) The hyperbola consists of two separate parts called branches.

b) The two fixed points are called foci. In the figure, the foci are at ( c,0).
c) The line containing the two foci is called the principal axis. In the
figure, the principal axis is the x-axis.
d) The vertices of a hyperbola are the points of intersection of the
hyperbola and the principal axis. In the figure, the vertices are at ( a,0).

e) The segment whose endpoints are the vertices is called the transverse axis. In the figure V 'V is the transverse
axis.

f) The line segment with endpoints (0,b) and (0,-b) where b  c  a is called the conjugate axis, and is a
2 2 2

perpendicular bisector of the transverse axis.


g) The intersection of the two axes is the center of the hyperbola .

h) The chord through a focus and perpendicular to the transverse axis is called a latus rectum. In the figure, G' G

b2 b2 2b 2
is a latus rectum whose endpoints are G’ (-c,  ) and G(-c, ) and has a length of .
a a a

3. The Asymptotes of a Hyperbola

Shown in the figure on the right is a hyperbola


with two lines as extended diagonals of the
rectangle shown.

These two diagonal lines are said to be the asymptotes of the curve, and are helpful in sketching the graph
x2 y2 b b
of a hyperbola. The equations of the asymptotes associated with 2
 2  1 are y  x and y   x .
a b a a
2 2
y x a a
Similarly, the equations of the asymptotes associated with 2  2  1 are y  x and y   x .
a b b b

y y

(6,9)

(-9,6) F(0,6) (9,6)

3y  x  0

F’(-6,0) (-3,0)O (3,0) F(6,0) x (0,3)

O x
(0,-3)
3y  x  0
(6,-9)
F’(0,-6)
x2 y2 y2 x2
The graph of  1. The graph of   1.
9 27 9 27

PRACTICE EXERCISES

Directions: Choose the best answer from the choices given and write the corresponding letter of your choice.

For items 1-5, use the illustration on the right.


3
M
1. Which of the following are the coordinates of A? 2
a. (1,2) b. (2,1) c. (-3,3) d. (2,-3) x 1 A
2. What is the distance between points M and T? -3 -2 -1 1 2 3
-10
a. 61 units b. 6 sq. units c. 51 units d. 8 units H
3. Which of the following points has the coordinates (-3,-1) -3
-2
a. M b. A c. T d. H T
4. Which of the following is the area of the triangle formed with vertices M, A and H? y
a. 5 sq. units b. 10 sq. units c. 5 units d. 10 units
5. Which of the following is the equation of the line containing points A and T? O(0,b)
a. y= 2 b. x=2 c. y+2x=3 d. y-2x+3=0
x
6. Suppose that an isosceles trapezoid is placed on the Cartesian plane as shown 0 D(a,0) E(b,0
On the right, which of the following should be the coordinates of vertex V?
a. (a,b) b. (b+a, 0) c. (b-a,b)d. (b+a,b)
7. The points (-11,3), (3,8) and (-8,-2) are vertices of what triangle?
a. Isosceles b. Scalene c. Equilateral d. Right

8. What is the area of the triangle in #7?


a. 40.5 sq units b. 41.8 sq units c. 42 sq units d. 46.8 sq units
9. Which of the following sets of points lie on a straight line?
a. (2,3), (-4,7), (5,8) b. (-2,1), (3,2), (6,3) c. (-1,-4), (2,5), (7,-2) d. (4,1), (5,-2), (6,-5)
3
10. If the point (9,2) divides the segment of the line from P 1(6,8) to P2(x2,y2) in the ratio r = , give the coordinates
7
of P2.
a. (16,–12) b. (–16, 15) c. (14,15) d. (12,–12)
11. Give the fourth vertex, at the third quadrant, of the parallelogram whose three vertices are (-1,-5), (2,1) and
(1,5).
a. (-3,-2) c. (-3,-4) c. (-4,-1) d. (-2,-1)
12. The line segment joining A(-2,-1) and B(3,3) is extended to C. If BC = 3AB, give the coordinates of C.
a. (17,12) b. (15,17) c. (18,15) d. (12,18)
0
13. The line L2 makes an angle of 60 with the L1. If the slope of L1 is 1, give the slope of L2.
a. (3 + 20.5) b. (2 + 20.5) c. –(2 + 30.5) d. –(3 + 30.5)
0
14. The angle from the line through (-4,5) and (3,m) to the line (-2,4) and (9,1) is 135 . Give the value of m.
a.7 b. 8 c. 9 d. 10
15. Which equation represents a line perpendicular to the graph of 2x + y = 2?
a. y = -0.5x – 2 b. y = –2x + 2 c. y = 2x – 2 d. y = 0.5x + 2
16. Which of the following is the y – intercept of the graph 2x – 2y + 8 = 0?
a. -4 b. -2 c. 2 d. 4
17. Which of the following may be a graph of x – y = a where a is a positive real number?
a. b. y c. y d. y
y

x x x x

18. Write an equation in standard form for a line with a slope of –1 passing through (2,1).
a. x + y = –3 b. –x + y = 3 c. x + y = 3 d. x – y = –3
b.
For items 19-22, use the illustration on the right. y

19. Which of the following are the coordinates of A? 2 T


a. (1,1) b. (1,-1) A 1.5

c. (-1,1) d. (-1,-1) 1

20. What is the distance between points A and H?


0.5

a. 61 units b. 6 sq. units 0 x


-3.5 -3 -2.5 -2 -1.5 -1 -0.5 0 0.5 1 1.5 2 2.5 3 3.5

c. 51 units d. 8 units -0.5

21. Which of the following points has the coordinates (-2,-2)? -1

a. M b. A H
M -1.5
c. T d. H
-2

22. Which of the following is the equation of the given graph?


a. y  x  2 .  
b. y   x  2 .
2
c. y  x  2 .  
2
 
2
d. y   x2  2 .  
23. Which of the following is the equation of the line containing points M and T?
a. y= 2 b. x=2 c. y-2x-2=0 d. y+2x+2=0
24. What is the shortest distance of x  8 y from x  3?
2

a. 1 unit b. 2 units c. 3 units d. 8 units


2 2
y x
25. Which of the following is a focus of   1?
12 4
a. (0,-4) b. (-4,0) c. (0,4) d. (4,0)
2 2
x y
26. What are the x-intercepts of   1?
4 9
a. none b.  2 c.  3 d. 4

27. Which of the following is a graph of a hyperbola?


a. y
b. 15
y

15

10

10

0 x

x -30 -25 -20 -15 -10 -5 0 5 10 15 20 25 30


0
-30 -25 -20 -15 -10 -5 0 5 10 15 20 25 30
-5

-5

-10

-10
-15

-15

c. d.
y
2
y
2

1.5
1.5

1
1

0.5
0.5

0 x
0 x
-3.5 -3 -2.5 -2 -1.5 -1 -0.5 0 0.5 1 1.5 2 2.5 3 3.5
-3.5 -3 -2.5 -2 -1.5 -1 -0.5 0 0.5 1 1.5 2 2.5 3 3.5
-0.5
-0.5

-1 -1

-1.5 -1.5

-2 -2

28. Which of the following is an equation of an ellipse that has 10 as length of the major axis and has foci which are
4 units away from the center?
y2 x2 y2 x2 y2 x2 y2 x2
a.  1 b.  1 c.   1 d.  1
25 9 9 16 5 3 16 25

For items 29-31, consider the graph on the right.


29. Which of the following is the equation of y
the graph?
 25x 2  2500
2 10
a. 100 y
b. 100 x  25 y 2  2500 2

c. 100 y  25 x  2500
2 2
0 x
-30 -20 -10 0 10 20 30
d. 100 x  25 y  2500
2 2

30. What are the x-intercepts of the graph?


a. none b.  2 -10

c.  5 d.  10

31. What kind of figure is shown on the graph?


a. circle b. ellipse c. hyperbola d. Parabola

32. Which of the following is the center of the graph


y
shown on the right?
a. (0,0) b. (0,10) 10
c. (10,0) c. (0,-10)

33. Which of the following is a focus of the graph


0 x
shown on the right? -30 -20 -10 0 10 20 30
a. (0,0) b. (0,10)
c. (0,5) c. (0,-10)
-10

34. What is the area of the shaded region?


a. 4 units b. 4 square units
c. 16 units d. 16 square units
LICENSURE EXAMINATION FOR TEACHERS (LET)

Refresher Course

Content Area: MATHEMATICS


Focus: ADVANCED ALGEBRA
Prepared by: Daisy de Borja-Marcelino
Competencies: Show mastery of the basic terms, concepts and equations in Advanced Algebra involving radicals,
rational exponents and functions. Solve, evaluate and manipulate symbolic and numerical problems in above areas
by applying fundamental principles and processes.

KEY IDEAS
Some Helpful Tips in Answering the LET

1 Read the question/s or the items carefully and understand what they say.
2 Determine what is/are wanted or what is/are asked for.
3 Find out what is/are given and which data are needed to solve the problem.
4 Reason out what processes (operations) to apply and the order in which they are to be applied.
5 Summarize the problem by means of an open number sentence.
6 Compute carefully. Check each step in the computations.
7 Decide the reasonableness of the result.
8 Check the result by seeing to it that the result satisfies all the conditions of the problem.
RATIONAL EXPONENTS
n
If a is a real number and n is any positive integer, the symbol a denotes the nth power of a. The real
number a is called the base and n is called the exponent. In symbols,

a n  a  a  a  ...  a .
n factors

Examples: a) 3333  34 or 81. b)  2m3   2m 2m 2m.


4
1 1 1 1 1
c)        . d) 1.31.3  1.32 .
2 2 2 2 2
e) math  mathmath .
e) 2 m m  2 m .     2
2

Note that any base raised to the power of 1, is just the base. Moreover, any base raised to the power of 0
is 1, while 00 is indeterminate.
Examples: a) 9  1. b) 4  4.
0 1
c) (-m)1 = - m. d) z0 = 1.

e) h op e
2 3 0
 1. f)  38.230  1 . g)  j 2o3 y 5 1  j 2o3 y 5 . h)
1 0 1 1
4
m a  a.
4

Laws of Exponents

If a and b are real numbers and m and n are positive real numbers, then the following are true.
a m a n  a mn . a  n m
 a nm .
m
If mn and a  0 , then a  a mn .
n
abn  a nb n .
a
n
am 1 a an
If n  m and a  0 , then n  nm . If b  0 , then    n .
a a b b
an
If a  0 , then  a0  1 .
an
Examples: a) 4  2 3
 46. b) 2 2   2
3 2 32
 25  32. c) (3 x 4 )2 = 32 x 42.
32
2
 1. 35 32 1 1
52 25
d) 3 e) 2  3  3  27.
3
f). 5  3  33  3  .
3 3 3 27
Exercises
1. In the expression 8m5, 5 is called the ________.
A. base B. coefficient C. constant D. exponent
25 m 0

2
20 m n s  9 3 14 0

 
2. Evaluate .
5m 10s 6 n8m 10
0 0

C. m  20
2
A. 5 B. 23m12n4s-2 D. undefined
1

3. Anthony wrote 3a  4b  = (3a) + 4b 4 . Which of the following is his misconception?
1 1
4
4

A. It is possible to factor out the exponent.


B. It is possible to distribute exponents over a sum.
C. The exponents should be multiplied with the base.
D. The coefficient inside the parentheses should be added.
2
 5r 4  25r 6
 
4. Is  3  equal to ? Why or why not?
 3s  9s 5
A. Yes, for the exponents inside the parenthesis and the numerical coefficients should be added to 2.
B. No, for the exponents inside the parenthesis and the numerical coefficients should be subtracted from
2.
C. Yes, for the exponents inside the parenthesis should be added and the numerical coefficients should
be
raised to 2.
D. No, for the exponent inside the parenthesis should be multiplied by 2 and the numerical coefficients
should
be raised to 2.
x12
5. Explain why 3
is not equal to x4?
x
A. Because the exponents should be added.
B. Because the exponents should be multiplied.
C. Because the numerator should be divided by the denominator.
D. Because the exponent of the numerator should be subtracted by the exponent of the denominator.
6. Which of the following is the product of x5y3z and x2y4z2?
A. 2x102y122z2 B. x10y12z2 C. 2x72y72z3 D. x7y7z3
7. Give the product of 5a and a 3  2a 5  11a.
A. 5a  10a  55a C. 5a  10a  55a
4 5 2 3 6

B. 5a  10a  55a D. 5a  10a  55a


4 6 2 3 5

  
3 5 8
8. Marlon claims that 2 2 is equal to 2 . Is he correct? Why or why not?
A. Yes, for the exponents inside the parenthesis and the numerical coefficients should be added to 5.
B. No, for the exponents inside the parenthesis should be multiplied by 2 and the numerical coefficients
should be raised to 5.
C. No, for the exponents should be multiplied.
D. Yes, for the exponents should be added.

The mentioned laws of exponents also hold when m and n are positive rational numbers.
Examples: The following are true if there is no zero denominator.
 12  12  1 1
  23  53  2 5

7
     31  3 .   
  m   m  m .
2 2 3 3 3
a) 3 3 3 b) m
  
     
2 5
2 1 1 5 3 2
53  p7 
c) 1
5 3 3
5 . 3
d) 3
 p7 7
 p7 .
3 7
5 p

e  e23  23  e0  1 .


2

e)
3
f) 3x y 
2 3 4
 314 x 24 y 34  34 x8 y12 .
e3
2
2
art 5 . x  4  1  1  1
2 5y
 art  5
g)    h) .
xy5 x  4 x  4  4 x  4 x  2
2 11 y 11 y 5 y 6y 3y
 xy 
Negative Exponents
1
If a is a nonzero real number and n is any rational number, then a-n = .
an
To simplify algebraic expressions with negative exponents is just to express the given expression into an
equivalent quantity where the exponents become positive.
Examples: Simplify the following such that they only have positive exponents.
1
 
3
2m  4
1
2
a) 3 b) 7 4
c)
1 3
 
1 1
2m 4  1
1
Solution: a) 3 2
 . b) 7 4
 . c) .
2m
1 3 1
2 4 4
3 7
Examples: Simplify the following such that they only have positive exponents. Assume nonzero bases and no
denominator is zero.

16 2 m 4 a t 3
a) b) c) d)
4 4 m 3 a 2 t 3

16 2 m 4 at 3 m 4 g 3
 32
1
e) f) g)
5
h) x  22
4 4 m 3a 2t 3 g 1h 3
Solution:

a)
16
=
2
4 4  
 4  1 .
2 2 4
b)
m 4
= m
 4 3  1
 m 43  m 1  .
4 4 3
4 4 4 m m
a 1 2  t 3
c) 2
= a  a1 2  a 3 d) = 1.
a t 3
16 2 m 4 at 3 44 m3aa 2t 3 44 m3a 3t 3 a 3
e) =  4 43  .
4 4 m 3a 2t 3 162 m 4t 3 4 mt m
3 1 3
m 4 g 3 m4 g 3 1 1 3 1 3 g h g 4 h3
f) =     g  h =  .
g 1h 3 1 g 1 h 3 m4 m4 m4
5
1 1 3
2
 
1 ( 5 )( ) ( 3)( )
5

g)
5
 3 2
=  2
 2
  2 2
 3 .
 2

h) x  22 = 1
2 or
1
or 2
1
.
x  2 x  2x  2 x  4 x  4
1 1 1 1 1 1 1
Note that is equal to  but 2 is not equal to 2   .
x  2x  2 x  2 x  2 x  4x  4 x 4x 4
a
Whenever the exponent of a base is in rational form where b 0, the expression can always be expressed
b
in radical form.

RADICALS
1
n
If n is a positive integer and a is a real number for which a n
is defined, then the expression a is called
1
n
a radical, and a = a . n

n
The symbol is a radical sign, the number a is the radicand and n is the index of the radical a.
3
Examples: a) In the expression 5 , the number 3 is called the index and 5 is the radicand.
1
4
b) The expression 27 can be written as 27 .
4

When a radical notation has no index, it is understood that n=2 or we are going to extract the square roots
of the radicand.
1
Examples: a)
2
49  49. b) 32  2 3  3.

Simplified radicals

An expression with radicals is simplified when all of the following conditions are satisfied.
Exponents of the radicand and index of the radical have no common factor except 1.
The radicand has no factor raised to a power greater than or equal to the index.
All indicated operations have been performed (if possible).
No denominator contains a radical.
The radicand has no fractions.

m
Radical notation of a n

 a
m
n m n
If a is a real number, m is an integer and a is a real number, then a n =
n
= am .
Examples: Write each exponential expression using radical notation.
1 3 3
a) a 3 b) m 4
c) 5 2

 
1 3 3
3
Solution: a) a 3 =
3
a. b) m 4
=(
4
m ) 3 . c) 5 2 =
2
53 = 2 5 or 53 .

Examples: Write each radical expression using exponential notation and simplify.
a) 22 b)
3
 27 c)
3
x6

 
1 1 6
1
Solution: a) 22 = 22 . 2
b)
3
 27 = (27) 3 =  33 3  3 . c)
3 6
x = x = x2. 3

If n is an even positive integer, then


n
a n = a =  a and if n is an odd positive integer then n
an = a .

Examples: Simplify each of the following and give all the roots.
4
a) 625 b) 81 c) 169x 6 d) 3
27 x 6 y 9

Solution:
a) 625  252  25 . b)
4
81 = 4 34 =  3.
169x 6 = 132 x 3  =  13x3.
2
c)

d) 3
27 x 6 y 9 = 3 33 x 2   y 
3 3 3
=3x2y3.

For all real numbers a and b, and positive integers m and n for which the indicated roots are real numbers,
the following are true.

( a )( b) =
n n n
ab . m n
a  mn a .
n
a a
n  n
where b is not equal to zero.
b b

Examples: The following are true.


a) 3  5  15. b) 4
81x  24 81x  8 81x . c) 12  x 6 
 
 x 3 12 .
Examples: Simplify each of the following.
8 4
32 y 5
a) 3 b)
125 243
 8 3  8 3 (2)3  2 2
Solution: a) 3  3    .
125 53 5 5 5

32 y 5
= 
4
16  2 y 4 y

4
24  2 y 4 y

2 y 4 2 y =  2 y  4
2y 
=
2 y 4 54 y  
 
b) 4 .
243 4
81  3 4
3 3
4
3 3 4
3  3  3

Operations on Radical Expressions

Addition and Subtraction


Radicals with the same radicand and the same index are called like radicals. Like radicals are added or
subtracted by using the distributive property of real numbers. Moreover, only like radicals can be combined.
3
Examples: a) Give the sum of 7 , 3 3 7 , 2 3 7 and 4 3 7 .
b) Evaluate 10 15 +3 15 - 15 .
c) If the lengths of the sides of a triangle is 24 cm, 2 6 cm and 4cm, give its perimeter.

3
Solution: a) 7 +3 3 7 +2 3 7 + 4 3 7 =10 3 7 .
b) 10 15 +3 15 - 15 = 12 15 .
c) Given a triangle, its perimeter is determined by adding the lengths of its sides. Hence,
24 cm + 2 6 + 4cm = 4  6 cm + 2 6 cm + 4cm
=2 6 cm + 2 6 cm+ 4cm = 4 6 cm + 4cm.

Multiplication of Radicals
Note that
n
ab =
n
a nb allows multiplication of radicals with the same index.

Examples: Give the product of the following in simplest form. Take only the positive roots.

a)  6 8  9 10 b) 23 m2 (3 m  33 m2 ) c)
(3 7 + 8 ) (3 7 - 8 )

Solution: a)  6 8  9 10 =  69 8  10  54 80  54 16  5  544 5  216 5 .

b) 23 m2 [3 m  33 m2 ] = 23 m2m  2 33 m2m2  23 m3 -(6) 3 m4

=
2m  6m 3 m .

c) (3 7 + 8 ) (3 7 - 8 ) = 9 72  82 = 97  8 =63-8=55.


Exercises

1. In the expression 34 5 , 5 is called the ________.


A. base B. index C. radicand D. root
2
5
2. Write 3 as a radical expression.
2 5 3
A. 35 B. 32 C. 3 D. 25 3
3. Write 5
5 y 3 as an exponential expression.

A. 5y  B. 5y    D. 5y 


1 3
3 5 3 5 3 3 5
C. 5 5 y
4. The sides of a triangle measure 3 15 m, 5 15 m and 15 m. What is the perimeter of the triangle?
A. 5 15 cm B. 8 15 cm C. 10 15 cm D. 12 15 cm
5. The side of a square measures 4 3 m. Give its perimeter.
A. 4 3m B. 8 3m C. 16 3m D. 20 3m

FUNCTIONS
A relation is a set of ordered pairs (x, y) such that for every first element x, there corresponds at least one
y. The set of all first elements is called the domain of the relation, whereas the set of second elements is the
codomain of the relation.
A function is a relation such that for every first element x of the ordered pair (x, y), there corresponds a
unique second element y. The set of all first elements is called the domain of the function, whereas the set of second
elements is the range of the function.
Tests for a Function
There are some tests that can determine whether a relation is a function or not. We have a function if no
two pairs in the set consisting of ordered pairs have the same first components.

Examples: a) The relation {(Mr. Cruz, Mark), (Mr. Cruz, Mary), (Mr. Gonzales, Art), (Mrs. Tan, Alice), (Miss Peralta,
Niko)} is not a function because more than one ordered pair have the same first component- Mr. Cruz.
b) The relation {(-1,0), (0, 1), (1, 2), (2, 3), (3, 4)} has no ordered pair that has the same first component.
Thus, it is a function. Moreover, its domain is the set {-1,0,1,2,3} and its range is {0,1,2,3,4}.
c) Consider the relation {(-3, -9), (-2, -4), (-1, -1), (0, 0),(1, -1), (2, -4), (3, -9)}. This relation is considered a
function because it has no ordered pair that has the same first component.

A relation may be described by a set of ordered pairs. A function is described by a set of ordered pairs with
no two pairs having the same first components.
Example: The table below shows the relation of the distance traveled by a car for a given length of time.
d = rt
Number of Hours 1 2 3 4 5 6 7 8 9 10
Distance 60 120 180 240 300 360 420 480 540 600

It can be observed that the distance traveled depends upon the number of hours or time. We say that
distance is a function of time. The relation can be expressed as d = rt or d = 60t in this particular example.

Example: The area of a square is a function of the length of its side.


A = s2
Side 1 2 3 4 5
Area 1 4 9 16 25

If a relation described by an equation or defined by a rule, a functional relationship exists if a change in the
independent variable x causes a change in the dependent variable y.

Another method of showing the relationship between the elements of two sets is by means of an arrow diagram.

0 0
Example: An arrow diagram for the relation y = 8x, where x is in set
of whole numbers from 0 to 3, is shown on the right. 1 8

2 16

3 24
x

0 2
10
2
Example: An arrow diagram for the relation {(x, y) │y = x + 2} is -1
shown on the right where x = -3, -2,-1,0,1,2,3 and y = 2,3,6,11. 1 3

-2
2 6

-3
3 11

A relation described by an arrow diagram is a function if:

1 there exists a one-to-one correspondence between the elements of the two sets
2 there exists a many-to-one correspondence between the elements of the two sets.
Another method of identifying a function from a mere relation is through its graph. A graph of a relation is
a function if a vertical line is drawn through the graph will intersect the graph in no more than one point.
The most fundamental way to graph a function is to plot points. Once the behavior of the graph of the
function becomes familiar, graphing becomes easier. Note that in graphing functions, we include all possible real
numbers in the domain.
We may start by creating a table of values in order to find out the behavior of the function. It is very
important to choose different numbers to get a clear picture of the graph. That is, it is helpful to generate as many
points possible.
1
Example: Graph f  x, y  y  3x   .
2
1
Solution: We note that f  x, y  y  3x   can be expressed in terms of slope and m = 3 and intercept
2
b = 0.5 . By plotting of points, we consider first a table of values.
X -2 -1 0 1 2 3
f(x) -6.5 -3.5 2.5 5.5 8.5
-0.5

y
8

0 x
-14 -12 -10 -8 -6 -4 -2 0 2 4 6 8 10 12 14

-2

-4

-6

-8

Note that the when a vertical line is drawn through the graph, it will intersect the graph in no more than
one point. Hence, the given relation is a function.

Example: If we have f(x) = x, this function is called an identity function defined by f = {(x, y) │y = x}.
Let us graph the identity function by assigning to x the values 0, 1, 2. Hence, the corresponding values of y
are 0, 1 and 2 respectively. Moreover, the line is determined by the following points.
X -2 -1 0 1 2 3
f(x) -2 -1 0 1 2 3
y
8

We now have the graph on the right. 4

0 x
-14 -12 -10 -8 -6 -4 -2 0 2 4 6 8 10 12 14

-2

-4

-6

-8
If f is defined by f = {(x, y) │y = b} where the range of the function f consists of one number, then f is a
constant function whose graph is a straight line parallel to the x-axis.

Example: Graph f(x) = 4.

Solution: We now have first a table of values.

x -2 -1 0 1 2 3
f(x) 4 4 4 4 4 4

6 y

0 x
-7 -6 -5 -4 -3 -2 -1 0 1 2 3 4 5 6 7

-1

-2

-3

Non-example: Graph y  x.
2

Solution: We now have first a table of values.

x 0 1 1 2 2 3 3 4 4
y 0 1 -1 2 2 2 2
- 3 - 3
y
4

0 x
-3 -2 -1 0 1 2 3 4 5 6 7 8 9 10 11 12

-1

-2

-3

-4

Observe that the graph of the said set of points shows that if any vertical line drawn through the graph
intersects the graph at more than one point. Hence, it is not a function.

The graph of a relation shows a function if any vertical line drawn through the graph intersects the graph at
no more than one point.

Non-examples: The graphs below do not define functions.


5
y
y
2.5
4

2
3

1.5
2

1
1

0.5
0 x
-8 -7 -6 -5 -4 -3 -2 -1 0 1 2 3 4 5 6 7 x
0
-1 -4.5 -4 -3.5 -3 -2.5 -2 -1.5 -1 -0.5 0 0.5 1 1.5 2 2.5 3

-0.5
PRACTICE EXERCISES

Directions: Choose the best answer from the choices given. Write the corresponding letter of your choice.

1. In the expression 9m3, 3 is called the ________.


A. base B. coefficient C. constant D. exponent

 17  73 
2. Evaluate  7  7  ?
  
  
2 4
2 
A. 7 B. 7 C. 7 7
D. 7 7
2
 1 3
3. Which of the following is equal to  m 2 n 2  ?
 
 
3 2 3
A. 0 B. 1 C. mn D. m n
3
4
4. Which of the following is equal to 2 ?
3
A.
4
23 B. 24 C. 24 D. 23
3
5. Give the index of the expression 34m .
A. 1 B. 2 C. 3 D. 34 m
6. In the expression 888tm , 888tm is called the __________.
A. Index B. Radicand C. Radical sign D. Exponent
5
7. Which of the following is equal to ?
5
A. 0 B. 1 C. 5 D. 2 5
8. The length of a rectangle is 3 3 m and its width is 2 2 m. Which of the following is the area of the rectangle?
A. 10 m 2
B. 5 6 m2 C. 9 6 m2 D. 6 6 m2

9. Rosalinda was asked to get the perimeter of a rectangle whose width is 2 5 mm and whose length is 5 5 mm.
Which of the following should be her answer?
7 10 mm
A. B. 14 10 mm C. 14 5 mm D. 10 5 mm
10. Is the sum of 2 , 3 , and 7 equal to 42 ? Why or why not?
A. No, because the indices should be multiplied.
B. No, because the terms should not be combined for these are not like radicals.
C. Yes, because the radicands should be added and the indices should be copied.
D. Yes, because the terms have no coefficient and the radicands should be added.
3
11. Write as an exponential expression: 3x 2 .

A. 3x  B. 3x   
1
3x 3
2
2 3 2 3 2
C. 3 3x D.
12. The sides of a triangle measure 2 7 m, 3 7 m and 28 m. Give the perimeter of the triangle.
A. 5 7 m B. 7 7 m C. 9 7 m D. 35 7 m
13. The side of a square measures 5 22 cm. Give its perimeter.
A. 10 22 cm B.15 22 cm C. 20 22 cm D. 20 11 cm
14. A secretary can type 324k words in 32k minutes. How many words can she type in a minute?
23k 23
1 1
A. 3 22k
B. 3 26k
C.   D.  
3 3
3
8x7 y 6
15. Simplify .
27

A.
2 x y  x 
2 2 3
B. -
2 x y  2 2 3
x
C.
8x3 y 3 x
D.
2 x y 
2 3
x
3 3 27 3
3 4 3 5
16. Which of the following is the index of the expression ho pe ?
A. 1 B. 2 C. 3 D. 4
3
17. In the expression 8m , 8m is called the __________.
A. Index B. Radicand C. Radical sign D. Exponent
 4
 2

18. Which of the following is equal to 3 5  3 5 ?
  
  
6 6 6 8
10 10 5 25
A. 3 B. 6 C. 3 D. 3
4
5
3
19. Which of the following is equal to 4 ?
5
3
4 8
5 5
A. 0 B. 1 C. 3 D. 3

3
 23 13 
20. Evaluate  x y  .
 
 
5
A. 0 B. 1 C. x2 y D. x3 y

21. Which of the following sets of ordered pairs describes a function?


A. {(-1, 4), (2, 8), (2, 20), (9, 36), (-3, -12)}
B. {(24, 6), ( 20, 5), (16, 4), (12, 3)}
C. {(1, 2), (-1, 3), (1, 4), (2, 3), (2, 4)}
D. {(1, -1), (-1, 1), (4, -2), (4, 2), (9, 3), (9, -3)}
22. Which of the following sets of ordered pairs DOES NOT describe a function?
A. {(-1, 4), (2, 8), (5, 20), (9, 36), (5, -12)}
B. {(-3, -2), (-2, -1), (-1, 0), (0, 1)}
C. {(24, 6), ( 20, 5), (16, 4), (12, 3)}
D. {(1, 1), (1, 2), (1, 4), (2, 3), (2, 4)}
23. Which of the following sets of ordered pairs describes a function?
A. {(0.5, 4), (1, 8), (1.5, 20), (2, 36), (-1, -12)}
B. {(4, 6), ( 4, 5), (16, 4), (3, 3)}
C. {(-1, 2), (-1, 3), (1, 4), (2, 3), (2, 4)}
D. {(0.25, -1), (0.5, 1), (0.75, -2), (1, 2), (9, 3), (9, -3)}
24. Which of the following sets of ordered pairs DOES NOT describe a function?
A. {(1.5, 1), (2, 1), (2.5, 1), (3, 1), (3.5, 1)}
B. {(-3, -2), (-2, -1), (-1, 0), (0, 1)}
C. {(24, 6), ( 20, 5), (16, 4), (12, 3)}
D. {(9, 1), (9, 2), (9, 4), (9, 3), (9, 4)}
25. Which of the following describes a function?
A.

x 1 1 2 4 4 6 6
y -2 -4 -6 -8 -10 -12 -14
B.

x 1 3 5 7 9 11 13
y -2 4 -6 8 -10 12 -14
C.

x 1 2 3 1 2 3 4
y 2 4 6 8 10 12 14
D.

x 1 2 3 4 5 1 2
y 0.5 1 1.5 2 2.5 3 3.5

26. Which of the following describes a function?


A.

x -1 -2 -3 -4 -5 -6 7
y 2 4 6 8 10 12 14
B.

x -1 1 -2 2 3 3 4
y 2 4 6 8 10 12 14
C.

x 1 -2 3 -1 -2 -3 4
y -2 -4 -6 8 10 12 14
D.

x 1 2 3 4 5 1 2
y -0.5 -1 -1.5 -2 -2.5 -3 -3.5
27. Which of the following arrows or mapping diagrams DOES NOT specify a function?

A. B.

1 6
-2 9
2 7
-1 4
3 8
0 0
4 9

C. D.
-2 3 2
4
2
-3 9 6 3
3
-4 9
16 4
4
28. Which of the following arrows or mapping diagrams specifies a function?

A. B.

2 1
4 2 9 4
6 5

C. D.

4 1 1 8
2 4
9 4
2
0
C.16 7

29. Which of the following arrows or mapping diagrams DOES NOT specify a function?
A. B.

6 6
-1 9
7 7
0 4
8 8
1 0
9 9

C. D.

-2 -4 2
3
2
-3 9 -9 3
3
-4 -
12 4
4 16

30. Which of the following graphs is a function?

A. C.
3 y
y
2
2.5

1.5
2

1
1.5

0.5

1
0 x
-3.5 -3 -2.5 -2 -1.5 -1 -0.5 0 0.5 1 1.5 2 2.5 3 3.5
0.5
-0.5

0 x
-1 -3.5 -3 -2.5 -2 -1.5 -1 -0.5 0 0.5 1 1.5 2 2.5 3 3.5

-0.5
-1.5

-1
-2

-1.5

B. D.
y y
3
2.5

2
2

1.5

1
1

0.5 0 x
-3 -2 -1 0 1 2 3

0 x
-3.5 -3 -2.5 -2 -1.5 -1 -0.5 0 0.5 1 1.5 2 2.5 3 3.5
-1
-0.5

-1 -2

-1.5

-3

31. Which is NOT a graph of a function?


A. B. C. D.
100 30 4 30
80 25 25
2
60
40 20 0 20
20 15 -4 -3 -2 -1 0 1 2 3 4 15
0 -2
10 10
-6 -5 -4 -3 -2 -1
-20 0 1 2 3 4 5 6 -4
-40 5 5
-60 0 -6 0
-80 -6 -5 -4 -3 -2 -1 0 1 2 3 4 5 6 -6 -5 -4 -3 -2 -1 0 1 2 3 4 5 6
-5 -8 -5
-100
-120 -10 -10 -10

32. Which of the following is NOT a graph of a function?

A. y C.
2 2.5
y

1.5
1.5

1
0.5

0 x

0.5 -3.5 -3 -2.5 -2 -1.5 -1 -0.5 0 0.5 1 1.5 2 2.5 3 3.5

-0.5

x -1
0
-1.5 -1 -0.5 0 0.5 1 1.5
-1.5

-0.5 -2

B. D.
1 y
3 y

0.5
2

0 x
-1.5 -1 -0.5 0 0.5 1 1.5 1

-0.5
0 x
-3 -2 -1 0 1 2 3

-1
-1

-1.5
-2

-2
-3

33. Which of the following graphs represents a function?

A. 3 y
C.
3 y

2
2

1
1

0 x
0 x
-3 -2 -1 0 1 2 3
-3 -2 -1 0 1 2 3

-1
-1
C. 3 y
D. 5
y

2
4

1
3

0 x
-3 -2 -1 0 1 2 3 2

-1
1

-2
0 x
-3 -2 -1 0 1 2 3

-3

34. Which of the graphs below best represents the following scenario? Chok was at home when he decided to visit
his friend. After a few hours, he was already at his friend’s house which is a few kilometers away from his home. He
stayed there for a few hours and went back home.

A. C.

Distance Distance

Time
Time

B. D.

Distance Distance

Time Time
35. Which of the graphs below best represents the following scenario? Avel is running at a steady rate and then
comes to a hill, which causes him to run at a slower rate. Once he reaches the top of the hill, he runs down the hill
very fast. Upon reaching the bottom of the hill, he resumes his original pace.
A. B.

Speed Speed

Time
Time

C. D.

Speed Speed

Time Time
Content Area: MATHEMATICS
Focus: Probability and Statistics
Prepared by: Daisy de Borja-Marcelino
LET Competencies:

Counting Techniques

Experiment: any activity that can be done repeatedly (e.g. tossing a coin, rolling a die).

Sample space: the set of all possible outcomes in an experiment.


Example: In rolling a die, the sample space is S = {1, 2, 3, 4, 5, 6}.

Sample point: an element of the sample space.


Example: In rolling a die, there are 6 sample points.

Counting Sample Points

1. Fundamental Principle of Counting (FPC)


If a choice consists of k steps, of which the first can be performed in n 1 ways, for each of these the second
can be performed in n 2 ways, for each of these the third can be performed in n 3 ways…, and for each of
these the kth can be made in nk ways, then the whole choice can be made in n1n2n3…nk ways.
Example: In how many ways can two dice fall? Ans. : 6 · 6 = 36 ways
2. Permutation
Permutation is an arrangement of objects wherein the order is important.

a. Linear Permutation
If n objects are to be arranged r objects at a time, then the number of distinct arrangements is given by
𝑛!
nPr =
(𝑛−𝑟)!
, 𝑛 ≥ 𝑟.
Example: In how many ways can the first, second and third winners may be chosen with 10 contestants?
10!
10P3 =
(10−3)!
= 10 · 9 · 8 = 720 𝑤𝑎𝑦𝑠
b. Circular Permutation
If n objects are to be arranged in a circular manner, then the number of distinct arrangements is (n - 1)!

Example: In how many ways can 6 people be arranged around a circular table?
Answer: (6 - 1)! = 5!

c. Permutation with Repetitions

The number of permutations of n things of which n1 are one of a kind, n2 second of a kind, …, nk of a kth kind is
n!
n1!n2 !...nk !
Example: How many different permutations are there in the word WAGAYWAY if all letters are to be taken?
8!
Answer:
2 !3 !2 !
3. Combination

Combination is the arrangement of objects regardless of order. In other words, the order of arranging the objects is
not important. If n objects are to be arranged r at a time, the number of distinct combinations is given by:
𝑛!
nCr = , 𝑛 ≥ 𝑟.
𝑟!(𝑛−𝑟)!
Example: In how many ways can a committee of 3 be chosen from 7 persons?
7!
Answer:
3! 4 !
Probability

Probability: the likelihood of occurrence of an event.


If E is any event, then the probability of an event denoted by P(E) has a value between 0 and 1, inclusive. In
symbols,
0 ≤ P (E) ≤ 1
If P(E) = 1, then E is sure to happen.
If P(E) = 0, then E is impossible to happen.
Moreover, if the probability that E will not happen is P(E’), then P (E) + P (E’) = 1.

Theoretical Probability
Theoretically, the probability of an event E, denoted by P(E), is defined as
n( E )
P(E) =
n( S )
where n(E) = number of favorable outcomes
n(S) = number of possible outcomes

Exercises

1. A coin is flipped once.

a. How many possible outcomes are there?


b. What are these outcomes?
c. How many of these outcomes consist of a face facing up?
d. What is the probability that a tail faces up?
2. A spinner can land on any of the numbers 1 through 12 with equal likelihood. What is the probability that the
spinner lands on a/an
a. odd number? b. number greater than 12?
c. prime number? d. number divisible by 3 or 4?
e. number divisible by 3 and 4? f. number between 3 and 9?

3. A box contains cards printed with the letters of the word “PHILIPPINES,” so that “P” is one 3 cards, “H” is on one
card, and so on. A card is drawn from the box.
a. How many possible outcomes are there?
b. What are these outcomes?
c. Are these outcomes equally likely?
d. What is the probability that the card drawn is
i. a “P” ii. a “H”
ii. a “I” iv. not a “P”

STATISTICS
Statistics is the branch of mathematics used to summarize quantities of data and help investigators draw
sound conclusions. Its two main branches are descriptive statistics and inferential statistics.
A sample is a specified set of measurements or data, which is drawn from a much larger body of
measurements or data called the population.

Kinds of Sampling
1. Random sampling techniques are used to ensure that every member of the population has an equal chance of
being included in the sample. A random sample is said to be representative of the entire population. The two
methods of random sampling are lottery method and the use of the table of random numbers.
2. Systematic sampling is a technique which selects every nth element of the population for the sample, with the
starting point determined at random from the first n elements.
3. Stratified random sampling is a technique of selecting simple random samples from mutually exclusive groupings
or strata of the population.
Graphical Representations of Data
Graphs are used in mathematics to show relationships between sets of numbers. Graphs are useful in the
field of statistics because they can show the relationships in a set of data.
1. Histogram - a graphical picture of a frequency distribution consisting of a series of vertical columns or rectangles,
each drawn with a base equal to the class interval and a height corresponding to the class frequency. The bars of a
histogram are joined together, that is, there are no spaces between bars.
2. Bar Chart- uses rectangles or bars to represent discrete classes of data. The length of each bar corresponds to the
frequency or percentage of the given class or category. The categories are in turn placed in either horizontal
3. Frequency Polygon- a special type of line graph, where each class frequency is plotted directly above the midpoint
or class mark of its class interval and lines are then drawn to connect the points.
4. Pie Chart- an effective way of presenting categorized (qualitative) distributions, where a circle is divided into
sectors - pie-shaped pieces - which are proportional in size to the corresponding frequencies or percentages.
5. Pictogram- known as picture graph where picture symbols are used to represent values.
MEASURES OF CENTRAL TENDENCY

A measure of central tendency is a single, central value that summarizes a set of numerical data. It describes
a set of data by locating the middle region of the set.
Measures Definition How to find Advantages Disadvantages
of Central
Tendency
Mean The sum of the data Ungrouped data: A single, unique Not appropriate for
divided by the number value that is skewed distribution
of data ∑𝑥 representative of all as it is affected by
𝑥̅ =
𝑁
the scores extreme scores or
outliers
Grouped Data:
Stable from group
∑ 𝑥𝑓 to group
𝑥̅ =
𝑁
May be used in
further
computations
Median The middle number of Ungrouped data: More stable from Not necessarily
the set when the data group to group than representative of all
are arranged in The middle for the the mode scores
numerical order 𝑁+1
( ) 𝑡ℎ 𝑠𝑐𝑜𝑟𝑒
2
Appropriate for Unstable from
skewed distribution group to group
Grouped Data:
𝑛
− 𝑐𝑓
Cannot be used in
Mdn = L + ( 2 )𝑖 further analyses
𝑓

Mode The number that occurs Ungrouped data: Easy to obtain Not necessarily
most frequently in the The most frequent representative of all
data score scores

Grouped Data: Cannot be used in


The class mark of the further analyses
class interval with the
highest frequency
Example: The scores for five students on a quiz are 40, 20, 30, 25, and 15. To find the mean score for this group of
students, we first find the sum of the scores:

40+20+30+25+15=130

We then divide the sum by 5, the number of scores:


130
 26 .
5

Example: Determine the median for the set of scores


82, 81, 80, 87, 20

Solution: We arrange the scores in Example 1 in order fro lowest to highest:


20, 80, 81, 82, 87

The middle number in this arrangement is 81 and it is the median.


Exercises

1. The mean score on a set of 10 scores is 71. What is the sum of the 10 test scores?

2. The mean score on a set of 13 score is 77. What is the sum of the 13 test scores?

3. The mean score on four of a set of five scores is 75. The fifth score is 90. What is the sum of the five scores?
What is the mean of the five scores?

4. Two sets of data are given. The first set of data has 10 scores with a mean of 70, and the second set of data
has 20 scores with a mean of 80. what is the mean for both sets of data combined?

MEASURES OF VARIABILITY

A measure of variation or variability describes how large the differences between the individual scores. The
common measures of variability are range and standard deviation.

Measures Definition How to find Advantages Disadvantages


of Variation

Range The difference between the Ungrouped data: Easy to compute Unstable
highest score and the lowest R = HS - LS
score Gives a unique value Not representative
Grouped Data: of the set of data
Easy to understand
R = Upper Limit of
the Highest Class Not used in further
Interval - Lower computations
Limit of the Lowest
Class Interval
Standard The square root of the Ungrouped data: Most stable Affected by extreme
deviation variance of the set of data scores
∑(𝑥− 𝑥̅ )2 Gives a unique value
S=√ More difficult to
𝑛
Grouped Data: Most representative compute and
understand
∑ 𝑓(𝑥− 𝑥̅ )2 Used in further
S=√ computations
𝑛

Exercises

1. On a quiz, the following scores were made in a class of 10 students: 72, 83, 86, 97, 90, 70, 65, 71, 80, 86. For this
set of scores, give the
a. mean b. median
c. mode d. range
e. midrange f. standard deviation

2. Jao, Dhei, Nelfe, Dada and Ched are all in the same statistics class. Their scores for the first two exams in the class
are listed in the accompanying table. The first exam had a mean of 84 and a standard deviation of 6, whereas the
second exam had a mean of 78 and a standard deviation of 4.
__________________________________
Exam 1 Exam 2
__________________________________
Jao 84 78
Dhei 90 74
Nelfe 66 78
Dada 78 70
Ched 84 78
__________________________________

a. Who improved on the second exam?


b. Who improved the most on the second exam?
c. Who did not improve on the second exam?
d. Considering both exams, who did the poorest?
e. Who performed the same on both exams?

PRACTICE EXERCISES
1. In a political science survey, voters are classified into six income categories and five education categories. In how
many different ways can a voter be classified?
A. 11 B. 25 C. 30 D. 45
2. The number of permutations of the word “probabilities” is
A. 13 B. 2!3! C. 13! D. 13!/(2!3!)
3. The number of ways seven visitors can be seated on a round table is
A. 6! B. 7! C. 6!7! D. 6!/2
4. In a certain town, 40% of the people have brown hair, 25% have brown eyes, and 15% have both brown hair and
brown eyes. A person is selected at random from the town. If he has brown eyes, what is the probability that he
does not have brown hair?
A. 3/8 B. 2/5 C. ½ D. none of these
5. A die is tossed. If the number is odd, what is the probability that it is prime?
2 3 2 1
A. B. C. D.
3 4 5 3
6. An accounting professor can assign grades of A, B, C, D, or F to student’s examinations. In how many ways can the
professor assign grades to three different student examinations?
A. 120 ways B.125 ways C. 25 ways D. 50 ways
7. A bag contains 15 red beads, 30 white beads, 20 blue beads, and 7 black beads. If one of the beads as drawn at
random, what is the probability that it will be white or blue?
25 15 13 17
A. B. C. D.
36 42 40 52
8. A whole number is chosen at random from the whole numbers from 1 to 50. What is the probability that it is an
even square?
A. 0.07 B. 0.06 C. 0.02 D. 0.09
9. A pair of dice is tossed. If the numbers appearing are different, find the probability that the sum is even.

1 2 2
A. B. C. D. none of
2 3 5
these
10. If repetitions are not permitted, how many 3 digits number less than 400 can be formed from the digits 2, 3, 5,
6, 7 and 9?
A.30 B.126 C.40 D. 162
11. In how many different ways can a true-false test be answered if there are five items, assuming that the student
give an answer for each item.
2 5
A.5! B.2! C. 5 D. 2
12. There are five musical numbers in a program. The number of ways these numbers can be presented is
D. 5!/ 3!2!
5
A. 5 B.5! C. 2
13. The numbers of permutations of the word “probabilities” is
A.13 B. 13! C.2! 3! D. 13!/ 2!3!

14. The number of ways seven students can be seated on a round table is
A. 6! B.6! 7! C.7! D. 6! /2
15. How many different signals, each consisting of 6 flags hung in a vertical line, can be formed from 4 identical red
flags and 2 identical blue flags?
A. 6! B. 6  4! C.4! 2! D. 6! 4!2!
16. The probability of getting a black heart is
A.0 B. 1 4 C.1 D. 1 52
17. In a certain town, 40% of the people have brown hair, 25% have brown eyes, and 15% have both brown hair and
brown eyes. A person is selected at random from the town. I he has brown eyes, what is the probability that he does
not have brown hair?
A. 3 8 B. 1 2 C. 2 5 D. None of these
For items 18-20: The scores of 10 students in a Math quiz are as follows:

Student A B C D E F G H I J
Score 18 32 10 15 27 18 23 28 29 30

18. What is the median score?


A. 18 B. 20 C. 23 D. 25
19. What is the mode of the scores?
A. 18 B. 20 C. 23 D. 25
20. What is the range of scores?
A. 18 B. 22 C. 23 D. 25
21. Jojo aims to get an average of 90 in 5 unit tests. The results of the first four unit tests are as follows: 89, 92, 86,
and 91. What should his grade in the fifth unit be for his average to be at least 90?
A. 91 B. 92 C. 93 D. 94
LICENSURE EXAMINATION FOR TEACHERS (LET)

Refresher Course

Content Area: MATHEMATICS


Focus: Calculus
Prepared by: Daisy de Borja-Marcelino
LET Competencies:

 Solve for the roots of a given quadratic equation


 Solve problems on quadratic equations
 Determine an equation given a set of roots which are imaginary/complex numbers
 Perform operations involving exponential and logarithmic functions
 Solve for the solution set of a given inequality
 Determine the rth term of the expansion (a + b)n
 Solve problems involving variations
 Determine the number of positive and negative roots of a given polynomial

Equations
An equation that contains at least one variable is called an open sentence. Equations b & c above are
examples of open sentences. In equation b, only -1 makes the sentence true or satisfies the equation. However,
more than one number might satisfy an equation. For example, +2 and -2 satisfy the equation x  4  0 . Any
2

number that satisfies an equation is called a solution or root to the equation. The set of numbers from which you
can select replacements for the variable is called the replacement set. The set of all solutions to an equation is called
the solution set to the equation. To solve an equation means to find all of its solutions.

QUADRATIC EQUATION
An equation of the form ax2 + bx + c = 0 where a  0, a ,b, and c are constants, is a quadratic equation.

ROOTS OF QUADRATIC EQUATIONS


To solve a quadratic equation means to find the value of x (unknown) that will satisfy the given equation.
The values of x that will make the equation true are called the roots or solution of the quadratic equation.

Methods of Finding the Roots of a Quadratic Equation

1. Factoring (Use this method if the quadratic equation is factorable)

Example: Determine the roots of x2 -8x - 15 = 0

Solution: Factoring the left side of the equation,


(x - 5) (x – 3) = 0

Equating each factor to zero,


(x - 5) = 0 (x – 3) = 0
x=5 x=3
2. Quadratic Formula
 b  b 2  4ac
The quadratic formula is x 
2a
BINOMIAL FORMULA

To obtain the terms of the binomial expansion (a + b) n, we use the binomial formula:

n (n  1)a n  2 b 2 n (n  1)( n  2)a n 3 b 3


(a + b)n = a n  na n 1 b    ...  nab n 1  b n
2! 3!

THE rth TERM OF THE EXPANSION (a + b)n

n (n  1)( n  2)...( n  r  2)a n  r 1 b r 1


rth term =
(r  1)!

EXPONENTIAL FUNCTIONS

The exponential function f with base b is denoted by f(x) = bx,


where b > 0 , b  1, and x is any real number.

Properties of f(x) = bx
 f has the set of real numbers as its domain.
 f has the set of positive real numbers as its range.
 f has a graph with a y-intercept of (0,1).
 f is a one-to-one function.
 f has a graph asymptotic to the x-axis.
 f is an increasing function if b>1 and f is a decreasing function if 0<b<1.

An exponential function has a constant base and a variable exponent.

The Natural Exponential Function

For all real numbers x, the function defined by f ( x)  e is called the natural exponential function. Note
x

that e is an irrational number and its accurate value to eight places is 2.71828183.

EXPONENTIAL EQUATIONS
An equation where the unknown quantity appears in an exponent is called an exponential equation.

SOLVING EXPONENTIAL EQUATIONS


To solve an exponential equation is to find the value of the unknown quantity in the given equation.

LOGARITHMIC FUNCTIONS

For x > 0, b > 0, and b  1, we have y = log a(x) if and


only if ay = x.

Note:
1. If the base of the logarithm is not indicated it is understood that the base is 10.
2. If the base of the logarithm is the number e, then it is called a natural logarithm and it is written as f(x) = ln x.

Properties of f x   log b x

 f has the set of positive real numbers as its domain.


 f has the set of real numbers as its range.
 f has a graph with a x-intercept of (1,0).
 f is a one-to-one function.
 f has a graph asymptotic to the y-axis.
 f is an increasing function if b>1 and f is a decreasing function if 0<b<1.

Remarks

 Logarithmic functions are the inverse of exponential functions.


 We can use the rules of exponents with logarithms.
 The two most common logarithms are called common logarithms and natural logarithms. Common
logarithms have a base of 10, and natural logarithms have a base of e.
 Equation in exponential form can be rewritten in logarithmic form, and vice versa.

The exponential form of y  log b x is b y  x .

The logarithmic form of b y  x is y  log b x .

Example1: The exponential equation 72 = 49 may be written in terms of a logarithmic equation as log7 (49) = 2.

1 1
Example 2: The exponential equation 9-3 = 3
or may be written in terms of a logarithmic equation as log 9
9 729
 1 
  = -3
 729 

Basic Properties of Logarithms

Property 1 : loga (1) = 0 because a0 = 1.

Example 1: In the equation 220 = 1, the base is 22 and the exponent is 0. Remember that a logarithm is an exponent,
and the corresponding logarithmic equation is log22 (1) = 0, where the 0 is the exponent.

0
2 2
Example 2: In the equation   = 1, the base is and the exponent is 0. Remember that a logarithm is an
5 5
exponent, and the corresponding logarithmic equation is log 2 1 = 0.
5

Property 2: loga (a) = 1 because a1 = a

Example 3: In the equation 71 = 7, the base is 7, the exponent is 1, and the answer is 7. Since a logarithm is an
exponent, and the corresponding logarithmic equation is log7 7 = 1

Example 4: Use the exponential equation m1 = m to write a logarithmic equation. If the base m is greater than 0,
then logm (m) = 1.

Property 3: loga (a)x = x because ax = ax

Example 5: Since 92=92, we may write the logarithmic equation with base 9 as log 9 92 = 2.

Example 6: Since you know that 112=112, we may write the logarithmic equation with base 11 as log 11112 = 2.

INEQUALITIES
Any relation expressed using the symbols <, >, > or < is called an inequality.
An absolute inequality is an inequality which is always true. A conditional inequality is one which is true
only for certain values of the variable involved.
1. 4 > 3 is an absolute inequality
2. x > 3 is a conditional inequality

PROPERTIES OF INEQUALITIES
Let a, b, c, & d be real numbers. The following hold.
1. Trichotomy Property
a > b or a < b or a = b
2. a > b if a - b > 0
a < b if a – b < 0
3.
a. If a> 0 and b> 0, then a + b> 0 and ab>0.
b. If a < 0 and b < 0, then a+b< 0 and ab> 0

4. Transitivity

If a < b and b < c then a < c.

5. Addition Property

If a < b and c < d, then a + c < b+ d

6. Multiplication Property

If a < b and c > 0, then ac < bc


If a < b and c < 0, then ac > bc

SOLVING INEQUALITIES

To solve an inequality means to find the value of the unknown that will make the inequality true.

POLYNOMIAL FUNCTION
The function defined by the equation
f(x) = a0xn +a1xn-1 + a2xn-2 + . . .+ an-2x2 + an-1x + an

where n is a nonnegative integer and a0, a1, . . ., an are constants, a0,  0 is a polynomial function in x of
degree n. The zeros or roots of f(x) are the numbers that will make f(x) = 0.

The Number of Positive and Negative Roots of a Polynomial Function


If f(x) is a polynomial function with real coefficients, then the following are true.
 The number of positive real zeros of f(x) is either equal to the number of variations in sign in f(x), or to that number
diminished by a positive even integer.
 The number of negative real zeros of f(x) is either equal to the number of variations in sign in f(-x), or to that
number diminished by a positive even integer.
LICENSURE EXAMINATION FOR TEACHERS (LET)

Refresher Course

Content Area: MATHEMATICS


Focus: ARITHMETIC, NUMBER THEORY AND BUSINESS MATH
Prepared by: Daisy de Borja-Marcelino
Competencies:

1. Simplifying expressions involving series of operations


2. Solve problems involving
a. GFC and LCMF
b. prime and composite
c. divisibility
d. inverse and partitive proportions
e. compound interest
f. congruence
g. linear Diophantine Equation
3. Apply Euler’s function and theorems, or Fermat’s theorem in solving problem.

THE NUMBER SYSTEM


Example: Some examples of imaginary numbers are:  25,  49 , negative numbers , 3i, -7i.

Example: Simplify: 2 (3 + 2i) – 5 (4 – 6i)

Solution: 2 (3 + 2i) – 5 (4 – 6i) = ( 6 + 4i) – (20 – 30i) = 6 + 4i – 20 + 30i = -16 + 34i.

Rational numbers are numbers which can be expressed as quotient of two integers, or can be expressed as fractions
2
in simplest forms. Examples are 8, -3, 3.45, and .
3
Irrational numbers are numbers which cannot be expressed as fractions in simplest forms. Examples are 3,4 7
3
, , e and .
3
Set of Natural/Counting numbers: {1, 2, 3, 4, … }. This set contains the numbers that we use in counting; also called
natural numbers.
Set of Whole Numbers: { 0 , 1, 2, 3, …}. This set is the union of the number zero and the set of counting numbers.

Set of Integers: { … , -3, -2, -1, 0, 1, 2, 3, …}. This set is the union of the set of counting numbers, their negatives, and
zero

II. THE COUNTING NUMBERS

A. Divisibility. An integer is divisible by a certain divisor (also an integer) if it can be divided exactly by that divisor.
That is, the remainder is zero after the division process is completed.
To illustrate, the integer 12 is divisible by 1, 2, 3, 4, 6, and 12.

To determine whether the integer is divisible by a certain integer or not, you may use the following
divisibility rules.

An integer is divisible by
a) 2 if it ends with 0, 2, 4, 6, or 8. (Examples: 134 or 12 or 12,330 or 4)
b) 3 if the sum of the digits is divisible by 3. (Examples: 132 or 18 or 12,330 or 45)
c) 4 if the last two digits form a number which is divisible by 4. (Examples: 13,412 or 12,332)
d) 5 if it ends with 0 or 5. (Examples: 135 or 10 or 12,330 or 495)
e) 6 if it ends with 0, 2, 4, 6, 8 and the sum of the digits is divisible by 3.(Examples: 134 or 12)
f) 7 if the difference obtained after subtracting twice the last digit from the number formed by the remaining
digits is divisible by 7. (Examples: 14 or 364)
g) 8 if the last three digits form a number which is divisible by 8. (Examples: 24160 or 5328)
h) 9 if the sum of the digits is divisible by 9. (Examples: 9, 432 or 18,504 or 270)
i) 10 if it ends with 0. (Examples: 120 or 7, 890 or 1, 230)
j) 11 if the difference between the sum of the digits on the even powers of 10 and the sum of the digits on
the odd powers of 10 is divisible by 11. (Examples: 2123 or 2816 or 94369 or 36465)
k) 12 if it is both divisible by 3 and 4. (Examples: 413,412 or 112,332)
l) 15 if it is both divisible by 3 and 5. (Examples: 150 or 350)

Remarks: Divisibility rules for two or more relatively prime numbers (GCF is 1) may be combined to serve as a
divisibility rule for their product.

Example: The rules for 3, 4, and 5 may be combined to serve as the rule for their product which is 60 since 3, 4, and
5 are relatively prime.
Exercises: Put a check mark on the space provided for, if the integer on the first column divides exactly the integer on
the top row.

456 36,720 800,112 456 36,720 800,112


2 10
3 11
4 12
5 14
6 24
7 32
8 45
9 77

Even numbers are whole numbers which can be divided exactly by two whole numbers.

Odd numbers are whole numbers which cannot be divided exactly by two whole numbers.

Example: If n3 is odd, which of the following is true?


I. n is odd II. n2 is odd III. n2 + 1 is odd

A) II only C) I only
B) I and II only D) I and III only

Example: If x is an odd integer and y is an even integer, which of the following is an odd integer?
A. 2x-y C. x2 + 3y
2
B. x + y - 1 D. x - 1
B. Factors and Multiples. In the number sentence 2 x 3 = 6, the numbers 2 and 3 are called factors, while 6 is their
product. Or we say, 2 and 3 are divisors of 6. Moreover, we say that 6 is a multiple of 2 and 3.

Example: How many factors does 42 have?


A) 2 B) 4 c) 5 D) 16
Answer: (C). The factors of 42= 16 are {1, 2, 4, 8, 16}.
Example: What are the multiples of 6?
Answer: The multiples of 12 are {12, 24, 36, 48, …}

Exercises Fill in the blanks with either 7 or 42.


a. ______ is a factor of ______.
b. ______ is divisible by ______.
c. ______ is a divisor of ______.
d. ______ is a multiple of ______.
e. ______ divides _______.

C. Prime and Composite Numbers

Prime numbers are counting numbers that have exactly two factors in the set of counting numbers: 1 and itself.

Composite numbers are counting numbers that have more than two factors in the set of counting numbers.

The numbers 0 and 1 are special numbers. They are neither prime nor composite.

Example: What is the sum of prime numbers less than 15?


A) 4 B) 5 C) 6 D) 14
Answer: The number 2,3,5,7, 11 and 13 are prime number less than 15. Hence, the answer is C.

D. Prime Factorization. This is a process of expressing a number as product of prime factors.

Example: Express 24 as product of prime factors.


Solution: 24= 2 x 2 x 2 x 3 = 23 x 3 or 3 x 23 .

Fundamental Theorem of Arithmetic


Every composite whole numbers can be expressed as the product of primes in exactly one way (the order
of the factors is disregarded).

E. The Greatest Common Factor (GCF)

The GCF of two or more numbers is the largest possible divisor of the given numbers.

Example: Determine the GCF of 12 and 42.


Solution: 24 = 2 x 2 x 3
42 = 2 x 3 x 7
GCF: 2x3=6

Example: What is the greatest integer that can divide the numbers 18, 24 and 36?
Solution: 18 = 3 x 3 x 2
24 = 3 x 2 x 2 x 2
36 = 3 x 3 x 2 x 2
GCF: 3 x 2 = 6

F. Least Common Multiple (LCM). The LCM of two or more numbers is the smallest possible number that can be
divided by the given numbers.
Example: Give the LCM of 20 and 30.
Solution: 20 = 2 x 2 x 5 = 22 x 5
30 = 2 x 3 x 5
LCM: 22 x 3 x 5 = 60.

Example: What is the smallest integer that can be divided by the numbers 24, 36 and 54?
Solution: 24 = 2 x 2 x 2 x 3 = 23 x 3
36 = 2 x 2 x 3 x 3 = 22 x 32
54 = 2 x 3 x 3 x 3 = 2 x 33
LCM: 23 x 33 = 216
G. Relatively Prime. Two numbers are relatively prime if their GCF is 1. The numbers themselves may not be prime.
The numbers 12 and 49 are relatively prime.

Example: Which of the following pairs are relatively prime to each other?
A)15 and 36 B) 23 and 51 C) 231 and 27 D) 121 and 330

III. INTEGERS

Consecutive integers are two or more integers, written in sequence, in which each integer after the first is 1 more
than the preceding integer.
Examples: 1,2,3,4,5, 6
3, 4, 5, 6, 7, 8

4, –3, –2, –1, 0, 1, 2, 3
x, x+1, x+2, x+3, x+4, x+5

The absolute value of a number x, denoted by  x , is the undirected distance between x and 0 on the number line.

– – – – –
5 4 3 2 1 0 1 2 3 4 5

It is also defined as

x if x ≥ 0
x = – x if x < 0

Examples:
Evaluate each of the following.
a) │2│ = 2 c) │0│ = 0

b) │– 7│ = 7 d) – │–15│ = –15

A. Multiplication. The product of two integers with like signs is a positive while the product of two integers with
unlike signs is negative.

Example: (-4) x 7 = (-28) or (-4) (7) = (-28) or (-4)  7 = (-28)


Example: (-8) x (-5) = 40 or (-8) (-5) = 40 or (-8)  (-5) = 40

B. Division. The quotient of two integers with like signs is a positive while the quotient of two integers with unlike
signs is negative.

Example: (-72) (-8) = 9


Example: (-123)  3 = - 41

C. Addition. The sum of two integers with like signs is the sum of their absolute values with the common sign prefixed
before it.

The sum of two integers with unlike signs is the difference of their absolute values with the sign of the
integer with the larger absolute value prefixed before the difference.

Example: (-3) + (-23) = (-26)


Example: (-34) + 12 = (-22)

D. Subtraction. Express subtraction statements as addition statements and follow the procedure in addition. (That
is, change the sign of the subtrahend to its opposite, and proceed to addition.)
Example: (-12) – (-3) = (-12) + 3 = -9

Exercise: What number should


a) be added to (-12) to yield 26?
b) be subtracted from (-2) to yield 5?
c) be multiplied by (-4) to yield (-36)?
d) be divided by (-2) to yield 30?
E. P-E-MDAS. P-E-MDAS stands for “Parenthesis-Exponent-Multiplication Division Addition Subtraction.

When two or more operations are involved in a single expression, operations are performed in the order of
P-E-MDAS. That is, we perform first the operation inside the parenthesis (or any grouping symbol), then followed by
determining the power of the number which is raised to a given exponent, then followed by multiplication/division,
and lastly by the addition/subtraction.
Should there be multiplication and division only, perform the operation from left to right.
Should there be addition and subtraction only, perform the operation from left to right.

Example: Simplify 20 + 100  ( 5 – 63  32 + 12)

Solution: 20 + 100  ( 5 – 63  32 + 12)


= 20 + 100  ( 5 – 63  9 + 12)
= 20 + 100  ( 5 – 7+ 12)
= 20 + 100  ( (–2) + 12)
= 20 + 100  10
= 20 + 10
= 30.

1. Two bells ring at 5 P.M. For the rest of the day, one bell rings every half hour whereas the other rings every 45
minutes. When is the first time, on that same day, that both bells ring at the same time again?
a. 6:30 P.M. b. 8:30 P.M. c. 8:45 P.M. d. 9:00 P.M.

2. Which is true?
a. The set of prime factors of 6 is {1,2,3} c. All prime numbers are odd numbers.
b. The product of irrational and rational is irrational. d. 3.14 is a rational number.

3. Which of the two-digit numbers below when inserted in the blank will make 38__09 divisible by 3?
a. 98 b. 84 c. 34 d. 60

4. Which of the following number is divisible by 45?


a. 300,000,000,450 b.600,000,000,045 c. 100,200,600,090 d. 400,450,000,000

5. On its anniversary, a certain store offers a free sandwich for every 4 th customer and a free softdrink for every 6th
customer. After 75 customers, how many had received both free sandwich and softdrink?
a. 30 b. 18 c. 12 d. 6

IV. FRACTIONS

Kinds of Fractions
As to relation between the numerator and the denominator
a. Proper – the numerator is less than the denominator
b. Improper – the numerator is equal to or greater than the denominator

As to relation of the denominators of two or more fractions


1 3 4
a. Similar – the denominators are equal. Examples:, ,
5 5 5
3 5 5
b. Dissimilar – the denominators are not equal. Examples: , ,
4 8 6

Other classes
3 6 12
a. Equivalent – fractions having the same value. Examples: , , .
4 8 14
3 5
b. Mixed – composed of a whole number and a proper fraction . Examples: 2 , 5 .
4 8
Rules involving Zero
a. Zero numerator and non-zero denominator – the value is zero
b. Zero denominator – no value, undefined
c. Zero value – the numerator is zero

A. Multiplication of Fractions. Multiply numerator by numerator and denominator by denominator to get the
numerator and denominator respectively of the product
12 5 60 2
Example:   or .
25 6 150 5

B. Division of Fractions. Multiply the supposed dividend by the reciprocal of the supposed divisor.
36 6 36 10 360 12 2
Example:     = or 2 .
25 10 25 6 150 5 5
1 2 19 3
Exercises: Evaluate the following. a) 2  3 b) 2 5
4 5 21 20
1 7 5 1
c) 2  1 d) 3 1
3 18 15 6
D. Changing Dissimilar Fractions to Similar Fractions. Determine the LCM of the denominators. Then with the said
LCM as the denominator, express each fraction to its equivalent.

1 3 5
Example: Express , , to similar fractions.
3 4 6

1 4 4 3 3 9 5 2 10
Solution:  = ,  = and  =
3 4 12 4 3 12 6 2 12
4 9 10
Therefore, the similar fractions are , and .
12 12 12
E. Addition of fractions. Convert the fractions to similar fractions. Then add the numerators to obtain the numerator
of the sum and copy the denominator.
1 3 5
Example: Evaluate + + .
3 4 6

Solution: The LCM is 12, so convert the addends to similar fractions with 24 as the
denominator.
1 3 5 4 9 10 23 11
+ + =  +  or 1 .
3 4 6 12 12 12 12 12

F. Subtraction of Fractions. Convert the fractions to similar fractions. Then subtract the
numerators to obtain the numerator of the difference and copy the denominator.

7 1
Example: What number should be subtracted from to obtain ?
12 2

Solution: Let the desired number be x. Then, the equation is given by


7 1 7 1 7 6 1
-x=  x= - =  = .
12 2 12 2 12 12 12

G. Fraction as Part of a Whole


3
Example: What is of 28?
4
Solution: Let the desired number be m. Then, the equation is given by
3 3 28 84
M =  28     21 .
4 4 1 4

Example: What part of 24 is 4?


Solution: Let the desired number be q. Then, the equation is given by
q  24 = 4

4 1
 q= or .
24 6
H. Simplifying Fractions
A fraction is in simplest form if the numerator and the denominator are relatively prime (their GCF is 1).
Thus, to simplify fractions, multiply by the fraction whose numerator and denominator are the reciprocal of the GCF
of the numerator and the denominator of the given fraction.

12 2 12 2  6 2
Example: The simplest form of is because   .
18 3 18 3  6 3

I. Ordering Fractions
Two fractions are equivalent if their cross products are equal. Otherwise, that fraction the numerator of
which was used to get the greater of the two cross products is the larger fraction.

Exercises

1. A 100-m wire is cut into two parts so that one part is ¼ of the other. How long is the shorter piece of wire?
a. 120m b. 80m c. 25m d. 20m

2. Luis left ½ pan of a cake on the table. Dada ate ¾ of it. What fraction of cake was left?
a. 1/8 b. 3/8 c. ¼ d. ½

n
3. If 21 and are equivalent fractions, what is the value of n?
39 26
a. 13 b. 14 c. 20 d. 21
5 1
4. Mr. dela Cruz owned of a business. He sold of his share in the business at a cost of P1M. What is the total
8 5
cost of the business?
a. P 6M b. P7M c. P 8M d. P 9M
5. Arrange the fractions 5/8, 4/5, 3/4 in increasing order.
a. 5/8, 4/5, 3/4 c. 3/4, 4/5, 5/8
b. 4/5, 3/4, 5/8 d. 4/5, 5/8, 3/4

6. Which of these fractions has the largest value?


a. 3/5 b. 11/16 c. 7/10 d. 5/8
7. Mark spent his monthly salary as follows: 3/5 for food and allowances, 1/3 for his child’s education and house
rental. If his monthly salary is P15, 000, how much would he left at the end of the month?
a. P 1,000 b. P2,000 c. P5,000 d. P 14,000

8. Chedy and Dada run for President for their organization. Chedy got 1/3 of the votes. If Dada got 300 votes, how
many students voted for Chedy?
a. 900 b. 200 c. 150 d. 100

V. DECIMAL NUMBERS

A. The Decimal Numbers and the place value chart

The place value chart


Hundred Thousands
Hundred Thousand

Ten Thousandths
Thousandths
Hundredths
Ten Thousand

Thousands

Hundreds

Tenths
Ones
Tens

100 000 10 000 1 000 100 10 1 0.1 0.01 0.001 0.0001 0.00001

The number 0.8 is read as “eight tenths” and .214 as “two hundred fourteen thousandths”.
The number 0.8 is equal to .800.
The number 0.8 is greater than 0.214.
Exercise: a) Arrange the following decimal numbers in ascending order:
0.5, 0.343, 0.142, 0.5254

b) In 2.3456, what digit is in the thousandths place?

B. Addition and Subtraction of Decimal Numbers. Addition of decimals is done by writing them in a column so that
their decimal points are aligned. Thus aligned, digits with the same place values would be in the same column, and
the addends (or the minuend and the subtrahend) are added (or subtracted) as if they were whole numbers.
C. Multiplication of Decimal Numbers. To multiply decimals, multiply the numbers as if they were whole and so
place the decimal point in the result as to have as many decimal places in it as there are in the factors combined.
D. Division of Decimal Numbers.

To divide a
i. decimal by a whole number, do as in dividing whole numbers but writing the decimal point directly above
that of the dividend.
ii. number by a decimal, multiply both dividend and divisor by that power of ten such that the divisor becomes
the least whole number, and then proceed as in (i) above.

VI. CONVERSION

A. Fraction to Decimal. Divide the numerator by the denominator.

Exercises: Convert the following to decimal:


a) 3/5 = ______
b) 5/6 = ______
c) 7/8 = ______

B. Decimal to fraction

a) Terminating – multiply the number by a fraction (equal to one) whose numerator and denominator is a multiple
of 10 such that the numerator of the product is a whole number.

Example: Convert 0.15 to fraction.

100 15 3
Solution: 0.15  = 
100 100 20

b) Repeating decimal number

Example: Convert 0.5 to fraction


Solution: Let n = 0.5 = 0.5555555.

10 n = 5.5555555.
- n = 0.5555555.
-----------------------------------
9n= 5
5
n=
9
5
Hence, 0.5 is equal to .
9

Exercises

1. Jeepney fares are computed as follows: P7.50 for the four kilometers plus P0.50 for every additional kilometer
thereof. How much should Au pay for a ride that covers 10 kilometers?
a. P8.00 b. P9.50 c. P10.00 d. P10.50
2. Which of the following is 0.3 of ♠ ♠ ♠ ♠ ♠ ♠ ♠ ♠ ♠ ♠?
a. ♠ ♠ ♠♠ ♠ ♠ b. ♠ ♠ ♠ c. ♠ ♠ d. ♠
3. Which of the following is between 3 and 4?
15
a. 5 b. -3.5 c. π d.
2
4. Evaluate 14.8 + 3.95 + .003.
a. 5433 b. 753 c. 446 d. 18.753
5. Carmen bought 4 kilograms of rice at P31.45 per kilo and 6 kilograms of salt at P22.35 per kilo. If she gave a P1000
bill to the cashier, how much change did she get?
a. P8.00 b. P9.50 c. P120.10 d. P740.10
6. Each capsule of a certain commercial vitamins contains 0.6 mg of calcium. In how many pieces of capsules can
22.2 mg of calcium be distributed?
a. 8 b. 37 c. 50 d. 105
7. Which of the following is equal to 2.4545454545… ?
5 5 44 44
a. b. 2 c. d. 2
11 11 45 45

1 1 1
8. The expression + + is equal to _____________.
100 1000 25
a. 0.0051 b. 0.006 c. 0.51 d. 0.051
9. Which of these numbers is greater than ¼?
a. .04 b. (1/2)2 c. 1/8 d. 1/0.04

VII. PERCENT

Per Cent – literally meaning “per hundred”, it is one way of writing fractions in which the denominator which is
required to be 100 is written as “%”, and read as “per cent”.
3 75 1
Since 1 = 100% hence  = 75%
4 100 4

A. CONVERSION
Percent to Decimal Number. Divide the number by 100%. Note that 100% = 1.

Example: Convert the following to decimal:


a) 35% c) 8.37%
1 1
b) 6 % d) %
4 4
Solution: a) (35%) 100% = 0.35
b) (6 ¼ %)  100%  (6.25%)  100% = .0625
c) (8.37)  100% = .0837
d) (1/4%)  100%  (0.25%)  100% = .0025

Decimal Numbers to Percent. Multiply the decimal number by 100%. Note that 100% = 1.

Example: Convert the following to percent.


a) 0.25 b) 0.143 c) 2.03 d) .005

Solution: a) 0.25 x 100% = 25% b) 0.143 x 100% = 14.3%


c) 2.03 x 100% = 203% d) .005 x 100% = 0.5%

Exercises: Fill in the blanks so that the entries in each row are equal.
Fraction Decimal Percent
A 4/7
B 160%
C 0.95
D 6/11
E ½%
B. Percentage. Percentage is a percent of a given number. The given number is called the base. The percent is called
the rate.

Formula: Percentage (P) = Base (B) x Rate (R)

Percentage Percentage
Base  and Rate   100%
Rate Base

Example: What is 50% of 140?


Solution: N = 0.50 x 140 = 70.00 = 70

Example: 36 is 10% of what number?


36
Solution: 36 = 0.10 N  N  360
0.10

Example: 22 is what percent of 88?


Solution: 22 = P  88
22
 P  0.25  100%  25%
88
Example: Mr. Ballaran receives a 10% increase in his salary. With the increase, he now receives P22, 000.
How much is his monthly salary before the increase?
Solution: present salary = previous salary + increase
22, 000 = previous salary + (10% of previous salary)
22,000= previous salary (1 + .10)
22 000  1.1 = previous salary
20,000 = previous salary
C. Discount

The discussion on DISCOUNT is very similar with the discussion on PERCENTAGE.


Original Price/ Marked Price/ List price - as the Base
Rate of Discount - as the Rate
Discount - as the Percentage
Selling Price - Original Price minus Discount

Example: A skirt with an original price of P250 is being sold at 40% discount. Find its selling price.

Solution: S.P. = Original price - Discount


= 250 – (0.40 x 250)
= 250 - 100
= 150

Example: An item has a selling price of P 210.00. If the selling price is 70% of the original price, what is its
original price?
Solution: Selling price is 70% of the original price
210 = 0.70  O.P.
 O.P. = 210  0.70 = 300.
Therefore, the original price is P300.

Example: A shirt is being sold at P 199.95. If its original price is P 430, find the rate of discount.
Solution: Discount = O.P. – S.P.
= 430 – 199.95 = 230.05
230.05
Rate of Discount =  100%  53.5%
430

D. Simple Interest
 Interest (I) is the amount paid for the use of money or the money earned for depositing the money.
 Principal (P) is the money that is borrowed or deposited.
 Time (t) is the number of days/months/years for which the money is being borrowed/deposited and interest
is calculated.
I I I
I = Prt, P= t= r=  100%
Example: Give the simple rt Pr Pt
interest of P10,000 for three
years at 5.5% per year.
Solution: I=Prt
I = (10 000) (.055) (3)
I = P1 100.

Example: Determine the amount of the principal if the interest at 10% per annum after 8 months is
P3,600.
I 3600
Solution: I=Prt  P = = 45,000
rt (0.10)(8 / 12)

Compound Interest ( Final Amount = P[ 1 + r ]n )


Compound interest is different from simple interest because after the first interest calculation, the interest is
added to the principal, so interest is earned on previous interest in addition to the principal. Compound Interest rates
may be given as annual (1 time a year), semiannual (2 times a year), quarterly (4 times a year), monthly (12 times a
year), and daily (365 times a year).
Example: If P500is invested at 8% compounded semiannually, what will the final amount be after three years?
Final Amount = P[ 1 + r ]n = 500[ 1 + (8% / 2)]3 * 2 = 500[ 1 + 0.04 ]6
= 500[1.27]
= 635
Exercises

1. John bought a jacket for Php 850.00. If he was given a discount of 15%, what was the original price?
a. P8,500.00 b. P1,000.00 c. P900.00 d.P765.00
2. In a basket, there are 15 santol, 12 balimbing, and 3 durian. What percent of the fruits are durian?
a. 10% b. 12.5% c. 12% d. 15%
3. A certain mobile phone model was sold for P4,000 in 2000. Two years later, the same mobile phone model sold
for P2,800. What was the percent decrease of the price?
a. 15% b. 30% c. 20% d. 35%
4. If ♥♥♥♥ is 50% of a larger figure, which of the following is the larger figure?
a. ♥ b. ♥♥ c. ♥♥♥♥ d. ♥♥♥♥♥♥♥♥
5. A senior class of 50 girls and 70 boys sponsored a dance. If 40% of the girls and 50 % of the
boys attended the dance, approximately what percent attended?
a 44 b. 46 c. 42 d. 40
1
6. Which of the following is equal to 2 %?
2
5
a. 2.5 b. 0.25 c. d. 0.025
2
7. Sarah’s earning P 9,200 a month will receive a 15% increase next month. How much
will her new salary be?
a. P 10,500 b. P 10,530 c. P 10,580 d. P 10,560
8. How much is 37% of 80% of 24?
a. 7.1 b. 1.92 c. 19.2 d. 71
9. According to the latest survey, 60% of the cancer patients were smokers. If there were 180 smoking cancer
patients, how many cancer patients are there in all?
a. 90 b. 108 c. 240 d. 300
10. Which of the following is 70% of 50?
a. 7 b. 17.5 c. 35 d. 71
11. Twenty four is 12% of what number?
a. 40 b. 150 c. 200 d. 400
12. Thirty six is what percent of 90?
a. 32.4% b. 40% c. 45% d. 76%
13. In a mathematics test of 40 items, Mavic got 90%. How many items did Mavic get?
a. 7 b. 28 c. 36 d. 360
14. Mr. Mabini receives a 10% increase in his salary. With the increase, he now receives P13,200. How much is his
monthly salary before the increase?
a. P12 000 b. P 13, 500 c. 14, 100 d. P14, 520

15. According to the latest survey, 60% of the cancer patients were smokers. If there were 180 smoking cancer
patients, how many cancer patients are there in all?
a. 70 b. 150 c. 300 d. 360
VIII. RATIO AND PROPORTION

 A ratio is a comparison of two or more quantities.

 A proportion is a number sentence stating the equivalence of two ratios.

Note that in ratio, we are comparing quantities of the same units and that the ratio is expressed in terms of
integers.
Examples: a) The ratio of 12 days to 3 weeks is 12:21 or 4:7.
b) The ratio of 3 meters to 180 cm is 300:180 or 5:3.
c) The ratio of 2 hours to 25 minutes is 120:25 or 24:5.
d) The ratio of 1 ½ to 4 ½ is 1:3.

A. Direct Proportion. As one quantity increases, the other increases also.

Example: Find the value of x if 15:20 = 14 : x.


Solution: Equate the product of the means and the product of the extremes. Then solve for x. Thus,

(15) (n) = (20) (14)

x
20 14 = 2
18.6 or 18 .
15 3

Example: A car travels at an average rate of 260 km in 5 hours. How far can it go in 8 hours,
if traveling at the same rate?
Solution: 260 : 5 = x : 8  (5) x = (260) (8)

x
260 8 = 416.
5
Example: If the ratio of teachers to students in a school is 1 to 18 and there are 360 students, how many teachers
are there?
Solution: Let x be the number of teachers,
1 x
 or 1 : 18 = x : 360
18 360
(18)x = (360)1
x = 360/18 = 20 teachers

B. Inverse Proportion. As one quantity increases, the other decreases.

Example: If the food is sufficient to feed 10 flood victims in 15 days, how many days would it last for 8 flood
victims?

Solution: Equate the product of the terms in the first condition to the product of the terms in
the second condition. Thus, we have:

(10 victims)(15 days) = (x) (8 victims)


x
1015 = 18.75 days
8

C. Partitive Proportion. One quantity is being partitioned into different proportions.

Example: A wood 120 m long is cut in the ratio 2:3:5. Determine the measure of each part.
2 120 24
Solution: · = = 24 m
10 1 1

3 120 36
· = = 36 m
10 1 1

5 120 60
· = = 60 m
10 1 1

Example: A wire is cut into three equal parts. The resulting segments are then cut into 4, 6, and 8 equal parts
respectively. If each of the resulting segments has an integer length, what is the minimum length of the wire?
A) 24 B) 48 C) 72 D) 96

Solution: Each third of the wire is cut into 4,6 and 8 parts respectively, and all the resulting segments have integer
lengths. This means that each third of the wire has a length that is evenly divisible by 4, 6, and 8. The smallest
positive integer that is divisible by 4, 6, and 8 is 24, so each third of the wire has a minimum length of 24. So, the
minimum length of the whole wire is three times 24, or 72.

Exercises
1. A 300 m ribbon is cut into four pieces in the ratio 1:2:3:4. Give the length of the shortest piece.2. If there are 18
boys and 45 girls in the gym, what is the ratio of the girls to the boys?
a. 2:5 b. 2:3 c. 5:2 d. 3:7
2. What one number can replace x in 2: x = x: 32?
a. 2 b. 6 c. 4 d. 8
3. If 5 men can do a job in 12 days, how long will it take 10 men to complete this task, assuming that they work at
the same rate?
a. 20 days b. 6 days c. 2 days d. 0.06 day
4. If 3 kg of oranges cost as much as 5 kg of chicos, how many kg of oranges would cost as much as 60 kg of chicos?
A. 100 B. 36 C. 7.5 D. 4
5. If 2/5 mm in a map represents 120 km, how many km will be represented by 2 mm?
A. 600 km B. 300 km C. 96 km D. 24 km
6. In a Mathematics Club, the ratio of boys to girls is 3:5. If there are 240 members, how many are girls?
A. 90 B. 144 C. 150 D. 450
7. A photographer wishes to enlarge a picture 18 cm long and 12 cm wide so that it will be
36 cm wide. How long will the enlarged picture be?
A. 54 cm B. 72 cm C. 24 cm D. 6 cm
8. If 8 secretaries can type 800 pages in 5 hours, how long would it take for 12 secretaries to
type 800 pages at the same rate?
A. 7 1/2 hours B. 3 1/3 hours C. 10 hours D. 2 1/2 hours
THE THEORY OF CONGRUENCES

If a and b are integers, m a positive integer and m(a – b), we say that “a is congruent to b modulo m”.
In symbols, we write this as a  b (mod m). CONGRUENCE was invented by Karl Friedrich Gauss at the beginning
of the 19th century and is a convenient statement about divisibility.

The following are equivalent and may be used interchangeably.


 a  b (mod m).
 m  (a – b) or (a – b) is divisible by m.
 a = b + mk, k  Z.

Theorem: If a and b are integers and m a positive integer then a  b (mod m) if and only if a and b leave the same
remainder upon division by m.

Let m be a positive integer. A collection of m integers {a , a ,..., a } is called a complete residue system modulo m
1 2 n

if every integer b modulo m is congruent to exactly one of the elements in the collection.

Properties of Congruence
Congruence is an equivalence relation in the set of integers; that is, congruence is reflexive, symmetric and transitive
with respect to integers.
In the following, let a, b, c, and d be integers and m a positive integer.
 If a  b (mod m) then
 a+c  b+c (mod m).
 ac  bc (mod m).
 ar  br (mod m) where r is a positive integer.

If a  b (mod m) and c  d (mod m), then


 a + c  (b + d) (mod m).
 ac  (bd) (mod m).
 m
 If ac  bc (mod m), then a  b mod  where d = (c, m).
 d

The following are some applications of congruence.


a) Finding the units digit (or hundreds digits) of a very large number written in exponential form; and
b) Finding the remainder when a very large number is divided by another number.

Two of the most prolific mathematicians in Number Theory are Pierre de Fermat and Leonhard Euler.

FERMAT’S AND EULER’S THEOREMS

Theorem 5. (Fermat’s Little Theorem) Let p be a prime number and a  Z . If p does not divide a, then
ap – 1  1 (mod p) .

Theorem 6. (Fermat’s Second Theorem). Let p be a prime number and a  Z . If p and a are relatively prime, then
ap  a (mod p) .
DEFINITION OF  (m)

Let m be a positive integer greater than 1. The number of positive integers less than and relatively prime to
m is the value of Euler’s totient or  function at m and is denoted by  (m) .

Remarks: If p is prime, then  ( p) = p – 1.

Theorem 7. Euler’s Theorem: If n is a positive integer and the greatest common divisor of a and n is 1, then
 
a n   1 mod n .

LINEAR DIOPHANTINE EQUATIONS

 An equation in one or more unknowns having integral solutions is called a Diophantine equation, in honor of
Diophantus of Alexandria.

Theorem 8. Given two integers a and b where (a , b) = d. The linear Diophantine equation
ax + by = c has an integral solution if and only if dc.

Theorem 9. If the equation ax + by = c has a solution x = x0 , y = y0, then any other solutions can be expressed in
the form
b
x  x0  t , t  Z and
d
a
y  y0  t , t  Z.
d
Example: To determine the integral solution of 24x + 138y = 18, we note that since (24,138) = 6 and 618. Then we
know that it has solution. We now have the following.

138 = 5 (24) + 18
24 = 1 (18) + 6
18 = 6 (3).
Observe that,
6 = 24 – 1(18)
= 24 – [138 - 5 (24)]
= (-1)(138) + 6(24)

Moreover,
18 = 3(6)
= 3[(-1)(138) + 6(24)]
=(-3)(138) +(18)(24)

Thus, y0 = -3 and x0 = 18

Hence, the solution of the equation is of the form y = -3+23t and x = 18– 4t where t is an integer.
There are problems which can be solved using linear Diophantine equations as working equations.
The following steps may be used in solving word problems which involve linear Diophantine equations in two
unknowns/variables:
Step 1. Represent the unknown values using any two variables.

Step 2. Form the equation using the condition given in the problem.
Step 3. Solve the resulting linear Diophantine equation.

Step 4. Determine the solution/s to the problem using the results in step 3.

Theorem: (Wilson’s Theorem) If p is a prime, then (p-1)!  1mod p  .

Exercises

1. Which of the following is true?


A. 34  3mod 5 B. 54  5mod 25 C. 308  0mod 11 D. 3  4mod 15
2. Mavic argues that 15  9mod 6 . Is she correct? Why?
A. Yes, because 6 divides 15 - 9. C. No, because 6 does not divide 15 + 9.
B. Yes, because 6 divides 15 + 9. D. No, because 15  9  61 .
3. Which of the following is congruent to 11 modulo 13?
A. -7 B. -5 C. -2 D. 4
4. Which of the following must be the value of n if n6  1  0mod 7 ?
A. x is divisible by 7 C. x is relatively prime with 7
B. x is prime D. x is any integer greater than 7
105
5. What is the remainder when 5 is divided by 3?
A. 1 B. 2 C. 3 D. 5
100
6. What is the units digit of 3 ?
A. 1 B. 3 C. 5 D. 9
7. What is  12 ?
A. 1 B.4 C.6 D. 11

SET A Exercises. Choose the letter of the best answer.

1. What is the sum of the first four prime numbers?


a. 11 b. 26 c. 17 d. 28
2. Which of the following is NOT true about the sum of two consecutive positive odd integers?
a. it is even b. it is only divisible by 12
c. it is divisible by 4 d. it is always divisible by 1
3. In a sequence of starts and stops, an elevator travels from the first floor to the fourth floor and then to the second
floor. From there, the elevator travels to the third floor and then to the first floor. If the floors are 3 meters apart,
how far has the elevator traveled?
a. 21 m b. 24 m c. 28 m d. 32 m
4. An orange light blinks every 4 seconds. A blue one blinks every 5 seconds while a red one blinks every 6 seconds.
How many times will they blink together in two hours?
a. once b. 2 times c. 10 times d. 60 times
5. If one prime factor of 84 is 3, what are the other prime factors?
a. 2 and 3 b. 2 and 7 c. 3 and 5 d. 4 and 7

6. A television show reports the following temperature for 5 cities:


Beijing London Chicago Philippines Moscow
2 0C -6 0C 0 0C 300C -9 0C
Which city is the coldest?
a. Beijing b. Chicago c. London d. Moscow
7. If the sum of a certain number and 7 is divided by 4, the quotient is 3. What is the number?
a. 5 b. 12 c. 15 d. 18
8. Which of the following numbers has the largest value?
a. –22 b. –10 c. –75 d.3
9. Which of the following numbers has the least value?
a. –22 b. –10 c. –75 d.3
10. What is the difference in the elevation between the top of a mountain 51 meters above sea level and a location
28 meters below sea level.
a. 23 m b. 33 m c. 79 m d. 89 m
11. A pack of P50-bills is numbered from RV628 to RV663. What is the total value of the pack of bills, in pesos?
a. 35 b. 36 c. 1750 d. 1800
12. Simplify: [ 5  81  32 – 5  3 + 2] (42 – 23)
a. 15 ¼ b. 4 c. -6 d. – 15 1/4
2
13. If each container contains 5 kg of flour, how many kg of flour are there in 12 container?
3
b. 68 kg b. 70kg c. 72 kg d. 80 kg

14. Eighteen is 2/3 of what number?


a. 6 b. 12 c. 6 d. 27
15. What part of an hour has passed from 2:48 am to 3:20 am?
a. 7/8 b. 1/3 c. 8/15 d. 8/25
16. Clarita spent one-sixth of her money in one store. In the next store, she spent three times as much as she spent
in the first store. If she had 80 pesos left, how much money did she have from the start?
a. 240 pesos b. 252 pesos c. 300 pesos d. 360 pesos
17. Philip has obtained the following grades: 1.4, 1.7, 1.8 and 2.5. What must be his fifth grade so that his average is
1.7?
a. 2.1 b. 1.9 c. 1.5 d. 1.1
18. Out of the 20 numbers, 6 were 2.5’s, 4 were 3.25’s and the rest were 2.2’s. Give the arithmetic mean of the
numbers.
a. 2.5 b. 2.65 c. 10 d. 22
19. Ron bought X number of notebooks at P23.00 each, Y pad papers at 18.45 each, and Z ballpens at P8.25 each.
If he gave an amount of P1000 to the cashier, how much change did he receive?
a. P 434.25 c. 1000 – [(23.00)(X) + (18.45) (Y) + (8.25) (Z)]
b. P 334.25 d. none of these
20. A bag has a selling price of P60.00. If the selling price is 75% of the original price, what is its original
price?
a. P80 b. P120 c. P200 d. P280
21. Mr. de Borja, a store owner, advertises a polo-shirt originally sold for P200 for P170 only. What rate of discount
is he giving?
a. P 30 b. P15 c. 30% d. 15%
22. Ja bought an article for P400 and sold it for P500. What rate of profit did she enjoy in that deal?
a. P100 b. 100% c. 25 % d. 20%
23. The price of an item is increased by 70% and then offered at 40% discount. What happened
to the original price?
a. There is an increase of 30%. c. There is an increase of 2 %.
b. There is an increase of 28%. d. There is a decrease of 32%.
24. How much should Allan invest so that his money earns P2,250 deposited at 6% for 9 months?
a. P 50,000 b. 37,500 c. P 135 d. P 101.25
25. Dan sells a real estate. He receives a monthly salary of P10,000 plus a commission of 1/5 % of his net sales for
that month. Find his gross pay for a month during which his net sale is one million pesos.
a. P 2,000 b. P 12,000 c. P 200,000 d. P 210,000
26. There are 20 million Filipinos who are qualified voters. If 25% of the population are qualified voters, how many
are not qualified voters?
a. 80 million b. 60 million c. 15 million d. 5 million
27. Three cavans of rice for a family of six members last for 5 weeks. At this rate, how many weeks will 4 cavans of
rice last a family of 8 members?
A. 4 B. 5 C. 5 1/3 D. 6
28. If the assembly, ratio of boys to girls is 1:4. What percent of the assembly are the boys?
A. 10% B. 20% C. 25 % D. 80%
29. What is the remainder when 1130 is divided by 31?
A. 1 B. 2 C. 10 D. 101
2061
30. Which of the following is the remainder when 5 is divided by 7?
A. 1 B. 2 C. 3 D. 6
31. If y is the remainder when 47 is divided by 6, what is the remainder when 19 is divided by y?
A. 1 B. 2 C. 4 D. 5
32. Which of the following is a value of z such that the congruence 3z  1mod 13 is NOT true?
A. 9 B. 22 C. 30 D. 48
33. What is the remainder when 18! + 2 is divided by 19?
A. 0 B. 1 C.3 D. 97
34. A certain number of sixes and nines are added to give a sum of 126. If the numbers of sixes and nines are
interchanged, the new sum is 114. How many of each were there after the switch?
A. Ten sixes and 6 nines B. Four sixes and Twelve nines
C. Seven sixes and nine nines D. Six sixes and Ten nines

SET B Exercises. Choose the letter of the best answer.

1. How many prime numbers are less than 37?


a. 9 b. 10 c. 11 d. 12
2. In a series of card games. Marlon starts out with P200 and wins a total of P450. If he later loses P350, wins P60
and loses P150, how much cash does Marlon have?
a. 0 b. P150 c. P210 d. P300
3. Your score in a game is -6. How many points must you earn to get a score of 10?
a. -6 b 15 c. 16 D. 22
4. Arrange the fractions 5/12, 3/7, 2/5 in decreasing order.
a. 2/5, 5/12, 3/7 c. 3/7, 5/12, 2/5
b. 5/12, 2/5, 3/7 d. 2/5, 3/7, 5/ 12
1 3
5. Edwin, Doms and Lon weigh 45 kg. If Edwin and Doms weigh 11 kg and 13 kg, respectively, what is the
5 4
weight of Lon in kilograms?
1 19 9 19
a. 20 b. 24 c. 42 d. 69
20 20 20 20
3
6. Which of the following should be multiplied to 4 so that the product is 57?
4
a. 6 b. 12 c. 6 d. 75
7. Which of the following is a value of m if 15  1mod m ?
A. 2 B. 3 C. 23 D. 32
1
8. Alex works on his assignment 3 hours a day, what part of the day does he spend doing his assignment?
3
a. 1/8 b. 1/7 c. 5/36 d. 15/28
9. What value of p will satisfy the equation 0.2 (2p + 1470) = p?
a. 294 b. 490 c. 560 d. 1470
10. A blouse originally priced at P600 is being sold at a discount of 30%. How much would you pay if you buy that
blouse?
a. P 30 b. P180 c. P 420 d. P 570
11. A pair of slippers with a selling price of P120 is sold at 40% discount. What is its original price?
a. P 48 b. P72 c. P 200 d. P 300
12. An item is offered at 20% discount. Later, it is offered at 30% discount. If the new selling price is P112, what is
the first original price?
a. P162 b. P200 c. P224 d. P1866.67
13. To have a 25% profit, the vendor should sell the item at P80.00. How much is his profit?
a. P20.00 b. P60.00 c. P16.00 d. P64.00
1
14. Minda deposited P50,000 in a bank that pays a simple annual interest of 14 %. How much
5
money will she have in the bank after five years.
a. P 85,500 b. P35,500 c. P36,250 d. P86,250
15. The ratio of cows and carabaos in the field is 4:9. If there are 468 cows and carabaos in the
field, how many are carabaos?
A. 52 B. 117 C. 144 D. 324
16. In the class, the ratio of boys to girls is 6:5. If there are 90 girls, how many persons are in the class?
A. 75 B. 108 C. 165 D. 198
17. Edwin painting a wall at 9:00 a.m. and was able to finish painting 3/5 of it at 10:30 a.m. Continuing at this rate,
at what time will he finish?
A. 10:45 a.m. B. 11:30 a.m. C. 11:45 a.m. D. 12:15 a.m.
200
18. What is the remainder when 4 is divided by 15?
A. -1 B. 1 C. 2 D. 4
19. If x is any positive integer, then 23x + 1 is _______ divisible by 8.
A. Always B. Never C. Sometimes D. Equivalently
20. What is  29 ?
A. 0 B. 1 C. 2 D. 28
21. Which of the following is the remainder when 2  1 is divided by 7?
50

A. 0 B. 2 C. 4 D. 5
22. Which of the following is equivalent to the pair of congruence x  1mod 4 and x  2mod 3 ?
A. x  1mod 12 B. x  2mod 12
C. x  3mod 12 D. x  5mod 12
23. Which of the following is NOT true if a is a positive integer?
C. a  amod 10
5
A. a divides a 5
B. a 5 divides a  1 . D. a and a 5 have the same units digit
24. Which of the following gives a remainder of 2 when divided by 5 and a remainder of 12 when divided
by 13?
A. 22 B.38 C. 77 D. 92
25. What is the remainder upon dividing the sum 2!4!6!...  2008!2010! by 5?
A. 1 B. 2 C. 3 D. 4
26. What is the least residue if 17109 is a multiple of 6?
A. 1 B. 2 C. 3 D. 5
27. Which of the following has an integral solution?
A. 21x  13mod 77 B. 3x  4mod 81
C. 3x  5mod 36 D. 3x  4mod 5
28. How many integral solutions does 18x  21mod 24 have?
A. 2 B. 3 C. 6 D. 10

29. Which of the following has a solution if the variables are positive integers?
A. 5x + 30y = 18 B. 8x + 10y = 15
C. 22x + 4y = 28 D. 101x  37 y  3819
30. When 16! is divided by 17, the remainder is ______.
A. 0 B. 1 C. 17 D. 18
31. A John’s transcript shows x number of 3-unit courses and y number of 5-unit courses for a total of 64 units. Which
of the following may appear in the transcript?
A. 2 x’s and 18 y’s B. 13 x’s and 5 y’s
C. 11 x’s and 3 y’s D. 9 x’s and 8 y’s

32. Which of the following is a value of x if x  1 mod 8 ?
2

A. 2 B. 5 C. 6 D. 10
33. When students in a certain college are grouped by 2’s, 3’s, 4’s, 5’s or 6’s at a time, there remain, 1,2,3,4, or 5
students respectively. When the students are grouped by 7’s, no is student left. What is the smallest possible number
of students in the school?

A. 227 B.1,534 C. 1,379 D. 2,778


LICENSURE EXAMINATION FOR TEACHERS (LET)

Refresher Course

Content Area: MATHEMATICS


Focus: Plane and Solid Geometry
Prepared by: Daisy de Borja-Marcelino

Basic Ideas

The undefined terms, point, line, and plane are geometric ideas and they are visually represented by a tiny
dot, a thin wire, and a smooth flat surface, respectively. Points are labeled by means of capital letters, lines by naming
any two of its points, and planes by naming at least three of its points. The subsets of a line are ray, segment, and
the line itself. Space is the set of all points.

A
.. B. A. B. A. B.
Line Ray Line segment
AB AB AB

Some postulates on points, lines and planes:

The Distance Postulate. To every pair of different points there corresponds a unique positive number.

The Ruler Postulate. The point of a line can be placed in correspondence with the real numbers in such a way that
(1) to every point of the line there corresponds exactly one real number;
(2) to every real number there corresponds exactly one point of the line; and
(3) the distance between any two points is the absolute value of the difference of the corresponding
numbers.

The Ruler Placement Postulate. Given two points P and Q of a line, the coordinate system can be chosen in such a way
that the coordinate of P is zero and the coordinate of Q is positive.

The Line Postulate. For every two points there is exactly one line that contains both points.

The Plane Postulate. Any three points lie in at least one plane, and any three noncollinear points lie in exactly one
plane.

The Plane Separation Postulate. Given a line and a plane containing it. The points of the plane that do not lie on the
line form two sets such that
(1) each of the sets is convex, and
(2) if P is in one of the sets and Q is in the other, then the segment PQ intersects the line.

The Space Separation Postulate. The points of space that do not lie in a given plane form two sets, such that
(1) each of the sets is convex, and
(2) if P is in one of the sets and Q is in the other, then the segment PQ intersects the plane.

The following are also true.


(a) Every plane contains at least three noncollinear points.
(b) Space contains at least four noncoplanar points.
(c) If two points of a line lie in a plane, then the line lies in the same plane.
(d) If two different planes intersect then, their intersection is a line.

Some theorems on points, lines and planes:

 If A,B, and C are three different points of the same line, then exactly one of them is between the other two.
 The Point-Plotting Theorem. Let AB be a ray, and let X be a positive number. Then there is exactly one point P
of AB such that AP  x .
 Every segment has exactly one mid-point.
 If two different lines intersect, their intersection contains only one point.
 If a line intersects a plane not containing it, then the intersection contains only one point.
 Given a line and a point not on the line, there is exactly one plane containing both.
 Given two intersecting lines, there is exactly one plane containing both.
 The First Minimum Theorem. The shortest segment joining a point to a line is the perpendicular segment.
 In a given plane, through a given point of a given line, there is one and only one line perpendicular to the given
line.
 The Perpendicular Bisector Theorem. The perpendicular bisector of a segment, in a plane, is the set of all points
of the plane that are equidistant from the end points of the segment.
 Through a given external point there is at least one line perpendicular to a given line.
 Through a given external point there is at most one line perpendicular to a given line.
 If M is between A and C on a line L, then M and A are on the same side of any other line that contains C.
 If B and C are equidistant from P and Q, then every point between B and C is equidistant from p and Q.
 If a line is perpendicular to each of two intersecting lines at their point of intersection, then it is perpendicular to
the plane that contains them.
 Through a given point of a given line there passes a plane perpendicular to the given line.
 If a line and a plane are perpendicular, then the plane contains every line perpendicular to the given line at its
point of intersection with the given plan.
 Through a given point of a given line there is only one plane perpendicular to the line.
 The Perpendicular Bisecting Plane Theorem. The perpendicular bisecting plane of a segment is the set of all points
equidistant from the end points of the segment.
 Two lines perpendicular to the same plane are coplanar.
 Through a given point there passes one and only one plane perpendicular to a given line.
 Through a given point there passes one and only one line perpendicular to a given plane.
 The Second Minimum Theorem. The shortest segment to a plane from an external point is the perpendicular
segment.
 Two parallel lines lie in exactly one plane.
 In a plane two lines are parallel if they are both perpendicular to the same line.
 Let L be a line and let P be a point on L. Then there is at least one line through P, parallel to L.

Angle. It is a union of two non-collinear rays that have a common endpoint.

An acute angle measures more than 0o but less than 90o.


A right angle measures exactly 90o.
An obtuse angle measures more than 90o but less than 180o.
Two angles are complementary if the sum of their measures is 90o.
Two angles are supplementary if the sum of their measures is 180o.
Two angles are adjacent if they have a common vertex, common side but no common interior points.
Two angles form a linear pair if they are both adjacent and supplementary.

Some postulates on angles:

 The Angle Measurement Postulate. To every angle there corresponds a real number between 0 and 180.
 The Angle Construction Postulate. Let AB be a ray on the edge of the half-plane H. for every number r between
0 and 180 there is exactly one ray AP , with P in H, such that mPAB  r .
 The Angle Addition Postulate. If D is in the interior of BAC , then mBAC  mBAD  mDAC .
 The Supplement Postulate. If two angles form a linear pair, then they are supplementary.

The transversal line is a line intersecting two or more coplanar lines at different points

1 2 Interior angles:  3,  4,  5,  6
3 4 Exterior angles:  1,  2,  7,  8
5 6 Corresponding angles:  1 and  5,  2 and  6
7 8  3 and  7,  4 and  8
Alternate interior angles:  3 and  6,  4 and  5
Alternate exterior angles: 1 and  8,  2 and  7
Same-side interior angles:  3 and 5,  4 and 6

Some theorems on angles and transversal lines:

 Given two lines cut by transversal. If a pair of alternate interior angles are congruent, then the lines are parallel.
 Given two lines cut by transversal. If a pair of corresponding angles are congruent, then a pair of alternate
interior angles are congruent.
 Given two lines cut by transversal. If a pair of corresponding angles are congruent, then the lines are parallel.
 Given two lines cut by transversal. If a pair of interior angles on the same side of the transversal are
supplementary, the lines are parallel.
 If two parallel lines cut by transversal, each pair of corresponding angles are congruent.
 In a plane, if a line intersects one of two parallel lines in only one point, then it intersects the other.
 Every right angle has measure 90, and every angle with measure 90 is a right angle.
 If two angles are complementary, then both are acute.
 Any two right angles are congruent.
 If two angles are both congruent and supplementary, then each is a right angle.
 The Supplement Theorem. Supplements of congruent angles are congruent.
 The Complement Theorem. Complements of congruent angles are congruent.
 The Vertical Angle Theorem. Vertical angles are congruent.
 If two intersecting lines form one right angle, then they form four right angles.
 The Angle Bisector Theorem. Every angle has exactly one bisector.
 The Isosceles Triangle Theorem. If two sides of a triangle are congruent, then the angles opposite these sides
are congruent.
 If two angles of a triangle are congruent, then the sides opposite them are congruent.
 If M is between B and C, and A is any point not on BC , then M is in the interior of BAC .
 The Exterior Angle Theorem. An exterior angle of a triangle is greater than each of its remote interior angles.

Polygon. It is a closed plane figure with three or more sides, consists of segments (sides) that meet only at their
endpoints (vertices) such that every side contains only two vertices, and every vertex is on exactly two sides.

Name of Polygon Number of Exact Sides Name of Polygon Number of Exact sides
Triangle 3 nonagon 9
Quadrilateral 4 decagon 10
Pentagon 5 undecagon 11
Hexagon 6 dodecagon 12
Heptagon 7 pentadecagon 15
Octagon 8

Notes: a) In a triangle, the sum of the lengths of two sides is always greater than the length of the third side.
b) In a right triangle with legs x, y and hypotenuse z: x2 + y2 = z2 (Pythagorean triple)
c) The sum of the interior angles of an n-gon is (n-2) 180o. The sum of the interior angles of a triangle is 180o
while the sum of the interior angles of a quadrilateral is 360o.

 A polygon is equilateral if the measures of all its sides are equal.


 A polygon is equiangular if the measures of all its angles are equal.
 A polygon is regular if all its sides and angles are correspondingly congruent.

Some postulates on polygons


 The SAS Postulate. Every SAS correspondence is a congruence.
 The ASA Postulate. Every ASA correspondence is a congruence.
 The SSS Postulate. Every SSS correspondence is a congruence.
 The Parallel Postulate. Through a given external point there is only one parallel to a given line.
 The Area Postulate. To every polygonal region there corresponds a unique positive real number.
 The Congruence Postulate. If two triangles are congruent, then the triangular regions determined by them have
the same area.
 The Area Addition Postulate. If two polygonal regions intersect only in edges and vertices (or do not intersect at
all), then the area of their union is the sum of their areas.
 The Unit Postulate. The area of a square region is the square of the length of its edge.
 The Unit Postulate. The volume of a rectangular parallelepiped is the product of the altitude and the area of the
base.
 The Cavalieri’s Principle. Given two solids and a plane. Suppose that every plane parallel to the given plane,
intersecting one of the two solids, also intersects the other, and gives cross sections with the same area. Then the
two solids have the same volume.
 The SAA Theorem. Every SAA correspondence is a congruence.
 The Hypotenuse-Leg Theorem. Given a correspondence between two right triangles, if the hypotenuse and one
leg of one of the triangles are congruent to the corresponding parts of the second triangle, then the
correspondence is a congruence.
 If two sides of a triangle are not congruent, then the angles opposite them are not congruent, and the larger angle
is opposite the longer side.
 If two angles of a triangle are not congruent, then the sides opposite them are not congruent, and the longer side
is opposite the larger angle.
 The Triangle Inequality. The sum of the length of any two sides of a triangle is greater than the length of the third
side.
 The Hinge Theorem. If two sides of one triangle are congruent, respectively, to two sides of a second triangle, and
the included angle of the first triangle is larger than the included angle of the second, then the third side of the
first triangle is larger than the included angle of the second.
 The Converse Hinge Theorem. If two sides of one triangle are congruent respectively to two sides of a second
triangle, and the third side of the first triangle is longer than the third side o the second, then the included angle
of the first triangle is larger than the included angle of the second.
 For every triangle, the sum of the measures of the angles is 180.
 Each diagonal separates a parallelogram into two congruent triangles.
 In a parallelogram, any two opposite sides are congruent.
 In a parallelogram, any two opposite angles are congruent.
 In a parallelogram, any two consecutive angles are supplementary.
 The diagonals of a parallelogram bisect each other.

Exercises

1. Which of the following has a definite length?


A) ray B) line C) angle D) line segment
2. A plane is determined by ______________.
i. a line and a point. ii. two intersecting lines.
iii. any three points. iv. a line and a point not on it.

A) ii only B) iv only C) ii and iii D) ii and iv


3. In angle LON, what is the vertex?
A) L B) O C) N D) cannot be determined

4. These shapes are arranged in a pattern.

Which of these shapes are arranged in the same pattern?

A)

B)

C)

D)

5. Which of these triangles cannot be isosceles?


A) acute B) right C) obtuse D) scalene

6. Two angles that are complementary _____________________.


i. are each acute. ii. are congruent.
iii. have equal measures. iv. have measures that add up to 900.
A) ii only B) iv only C) ii and iii D) i and iv
7. Refer to the figure on the right. Given: m  1 = 57° and m  3 = 80°. What is m  5?
A) 43° B) 63° C) 137° D) 180°

8. A quadrilateral MUST be a parallelogram if it has ________________________.


A) one pair of adjacent sides equal B) one pair of parallel sides
C) two pairs of parallel sides C) two adjacent angles equal
9. NCTE is a parallelogram. If m  N = 67°, then m  T = _____.
A) 113° B) 90° C) 67° D) 23°
10. Which two properties of the rhombus do not hold true for other parallelograms?
A) Diagonals are perpendicular and congruent.
B) Diagonals are congruent and bisect each other.
C) Diagonals are perpendicular and bisect each other.
D) Diagonals are perpendicular and each bisects the angles of the parallelogram.
11. The supplement of an angle is three times its measure. What is the measure of its supplement?
A) 22.5° B) 45° C) 67.5° D) 135°

12. Refer to the figure on the right. If lines r and s are parallel,
which of the following pairs of angles are congruent?

A)  4 and  5 B)  4 and  3 C)  1 and  8 D)  3 and  5

13. The angles of a triangle are in the ratio 2: 3: 5. What is the largest angle?
A) 36° B) 54° C) 90° D) 99°
14. An angle measures 65.5°. What is the measure of its supplement?
A) 24.5° B) 25. 5° C) 114.5° D) 124.5°
15. Which refers to a pair of lines that intersect and form 4 right angles?
A) parallel lines B) perpendicular lines C) intersecting lines D) skew lines
16. Which of the following quadrilaterals best describes a square?
A) Its diagonals are perpendicular to each other. B) It is an equiangular rhombus.
C) Its diagonals are congruent. D) It has four right angles.
17. If ABC is an isosceles triangle with a right angle at B, then
̅̅̅̅ is the hypotenuse.
A) 𝐵𝐶 B) AC = BC
C) angle BCA measures 45°. D) AB = ½ AC a
18. What is the measure of each interior angle of a regular pentagon?
A) 108° B) 140° C) 180° D) 540°
19. What is the sum of the measures of the interior angles of a regular heptagon? m
A) 1260° B) 1080° C) 900° D) 112.5° t h

20. In the adjoining figure, if m = 63° and h = 134°, then a must be equal to _____.
A) 46° B) 71° C) 109° D) 117°

21. Which of the following statements is true about parallel lines?


A) They form a right angle. B) They do not intersect at all.
C) They are skew. D) None of these.
22. In the adjoining figure, the quadrilateral is a parallelogram. What is the value of x?
A) 150 B) 75 C) 60 D) 50

23. Which of the following can be lengths of the sides of a triangle?


A) {3, 4, 9} B) {5, 5, 10} C) {-12, 6, 8} D) {3, 18, 20}
24. The measures of the exterior angles of a triangle are in the ratio 2:3:4. What is the measure of the widest angle?
A) 40° B) 80° C) 160° D) 170°
G. Perimeter. The distance around a polygon is called a perimeter. To obtain this, we just add the length of all the
sides of the polygon. In the case of equilateral polygons, we just multiply the length of one side to the number of
sides.

Example: Determine the perimeter of a triangle with sides 7, 12, 18.


Solution: P = 7 + 12 + 18 = 37 units.

Exercises: Complete the table below.

Regular Polygon Length of a Side Perimeter


a) square 13 cm
b) 7 cm 56 cm
c) nonagon 54 cm
d) 9 cm 45 cm
e) heptagon 28 cm

H. Areas of Triangles and Quadrilaterals

a) triangle = (1/2) (base) (height)


b) rectangle = (length) (width) or (base) height)
c) square = (side)(side) or (1/2)(d1)(d2)
d) parallelogram = (base) height)
e) rhombus = (1/2)(d1)(d2)
f) trapezoid = (1/2) (upper base + lower base)(height)

Example: Give the area of a square whose perimeter is 100 m?


Solution: Since the perimeter of the square is 100 cm, then each side measures 25 m. Hence, its area is (25
m)2 = 625 m2.
Example: Determine the area of a triangle whose base is 10 mm and its altitude is thrice its base.
1 1
Solution: A= bh  (10) [(3) (10)]  150 mm2 .
2 2

Example: Give the area of a rectangle whose width is 5 cm and its length exceeds 4 times the width by 3.
Solution: A = lw = [4w + 3] (w)
= [(4)(5) + 3] (5)
= (23) (5) = 115 sq. cm.

Example: Determine the area of the trapezoid whose bases are 6 cm and 10 m, while the altitude is 7 m.

Solution: A = (1/2) (b1 + b2) (height) = (1/2) (6 + 10) (7) = 56 m2

I. Circle. This is a set of points in a plane, equidistant from a fixed point. The fixed point is
called the center, and the fixed distance is the length of the radius.

 Circumference is the distance around the circle. Formula: 2 r


 Area is the interior region bounded by the circle. Formula:  r2
 Terms related to a circle:
Center – the fixed point
Radius – segment (or distance) drawn from the center to any point on the circle
Chord – segment whose endpoints are any two points on the circle
Diameter – a chord that passes through the center (the longest chord)
Secant – a line/ray/segment that cuts the circle in two points
Tangent – a line that touches the circle in only one point

Note: The diameter is twice as long as the radius.

Exercises: In the figure on the right, identify the following:


center, radius, chord, diameter, secant, tangent.

Example: Determine the circumference of a circle with radius 10 mm.


Solution: C = 2 r = 2 (3.1416) (10 cm) = 62.832 mm

Example: Give the area of the circle if its circumference is 18  m.


Solution: Since the circumference is 18 cm, then its radius is 9 m. Hence,
A =  r2 = (3.1416) (9m)2 = 254.4696 or 254.47 m2

Exercises
1. Complete the table below.
Radius Diameter Circumference (in  ) Area (in  )
1. 2 cm
2. 6 dm
3. 50  m
4. 36  km2
5. 9 m

2. A diameter of a circle is also a ________.


A) radius B) secant C) chord D) tangent
3. Which of the following sets of numbers can be sides of a right triangle?
A. 4, 5, 6 B. 6, 6,2 3 C. 3, 6,3 2 D. 2, 2, 3
4. The length and width of a rectangle are (3x – 2) and (2x + 1). What is its perimeter?
A. 5x-1 B. 10 x - 2 C. 6x2 –x -2 D. 6x2 –x +2

5. The area of a square is 32x. Which of the following could be the value of x?
A. 2 B. 6 C. 3 D. 4
6. If the area of one circle is twice of another circle, what is the ratio of the area in percent of the smaller to larger
circle?
A. 70% B. 25% C. 75% D. 50%

J. Volume of Solids. It describes how much space a three dimensional figure occupies.

a) cube = (side)3
b) rectangular prism = (length)(width)(height) or (area of the base)(height)
c) pyramid = (1/3) (area of the base) (height)
d) sphere = (4/3)( )(radius)3
e) cylinder =  (radius)2 (height)
f) cone = (1/3)  (radius)2 (height)

Example: What is the volume of a cube whose edge is 4 mm?


Solution: V = (side)3
= (4 mm)3
= 64 mm3

Example: The length of a rectangular box is 20 cm. Its width exceeds 1/4 of the length by 5
cm, while the height is 7 cm less than 1/2 of the length. What is its volume?

Solution: Volume = (length)(width)(height)


1  1 
= (20 cm)   20cm  5cm    20cm  7cm 
4  2 
= (20 cm) (10 cm) (3 cm) = 600 cm3

Example: The area of the base of a pyramid is 48 cm2 while the height is 6 cm. What is its
volume?

Solution: V = (1/3) (area of the base) (height)


= (1/3) (48 cm2) (6 cm ) = 96 cu. cm.
Example: Determine the volume (in ) of a sphere whose diameter is 12 m.
Solution: V = (4/3)( )(radius)3 and since diameter is 12 cm, then the radius is 6 m.
= (4/3)( )(6 cm)3 = 288  m3

Example: A cone has a base diameter of 32 cm and its height is 3 cm less than 3/4 of the
radius. What is its volume?

Solution: V = (1/3)  (radius)2 (height)


1 3 
=    (16 cm)2   16cm  3cm 
3 4 
= 768  cm3

K. Lateral and Total Surface Areas of Solids.

Solids Lateral area Total surface area


a) cube 4 s2 6s2
b) rectangular prism (perimeter of the base)x height 2B + ph
c) pyramid 1/2 (perimeter of the base) slant height B + 1/2 p (slant height)
d) sphere None 4r2
e) cylinder (Circumference of the base) (slant height) 2r2 + 2r (slant height)
f) cone 1/2 (Circumference of the base) (slant height) r2 + r (slant height)

Exercise

1. Determine the volume, lateral and surface area of the following: (All units are in cm.) Express your answers in ,
if possible.

edge = 2 mm
length = 10cm, base edge = 10 m
width = 3cm, slant height = 13m
height = 4cm altitude = 12m

radius = 5 radius = 5 radius = 5


height = 12 slant height = 5
altitude = 4
Volume (in cm3) Lateral Area (in cm2) Total Surface Area (in cm2)
a) cube

b) rectangular prism

c) square pyramid

d) sphere

e) cylinder

f) cone

PRACTICE EXERCISES

1. What will happen to the area of a circle if its radius is doubled?


a. The area will be doubled. c. The area will be quadrupled.
b. The area will remain the same. d. The area will be reduced to half.

2. What is the reason for your answer in item number 1?


a. The area of the circle varies directly to the radius.
b. The area of the circle varies inversely to the radius.
c. The area of the circle varies directly to the square of its radius.
d. The area of the circle varies inversely to the square of its radius.
3. If a side of square measures 5mm, which of the following is its perimeter?
a. 20mm b. 25mm c. 20 mm2 d. 25mm2
4. If a square has an area of 144m2, which of the following is the measure of its side?
a. 12mm b. 12cm c. 1,200 cm d. 1.2m
5. If the base of a triangle measures 20 centimeters and its height measures 1 inch, which of the following is
its area?
a. 20 cm b. 25.4 cm c. 20 cm2 d. 25.4 cm2
6. A string 1mm long is used to form a shape that would give a maximum area. What shape is it?
a. square b. rectangle c. circle d. parallelogram
7. If the length of the side of a square is reduced to half, what will happen to its perimeter?
a. It will be tripled. c. It will be doubled.
b. It will remain the same. d. It will be reduced to half.
8. The area of a new circle is nine times its original area. What made this happen?
a. The radius was tripled. c. The value of π was squared.
b. The radius was multiplied by 9. d. The radius was reduced to half.
9. A regular polygon is inscribed in a circle. Suppose we increase continuously the number of its sides, what
do you think will happen?
a. The regular polygon with infinitely many sides will never exist.
b. It is impossible for a regular polygon with infinitely many sides to be inscribed in a circle.
c. The perimeter of the regular polygon will be greater than the circumference of the circle.
d. The perimeter of the regular polygon will be approximately equal to the circumference of the
circle.
10. Which of the following is the most reasonable weight of an adult woman?
a. 12 lbs b. 120 lbs c. 1,200 lbs d. 12,000 lbs
11. Which of the following is the nearest approximation of 10 kilometers?
a. 5 miles b. 10miles c. 16 miles d. 20 miles
12. Consider a rectangle with a fixed perimeter. Assume that we allow continuous variations of either its width
of length. Which of the following is NOT possible?
a. The area of the new rectangle will be greater than the original area.
b. The area of the new rectangle will be smaller than the original area.
c. The area of the original rectangle will be equal to the area of all new rectangles that can be formed.
d. The area of the original rectangle will be equal to the area of some new rectangles that can be
formed.
13. What is the reason for your answer in item number 12?
a. Rectangles possibly have the same perimeter but of different area.
b. Rectangles with the same perimeter always have the same area.
c. If rectangles have larger perimeter, then their area is wide.
d. If rectangles have small perimeter, then their area is small.
14. A largest circle is to be made out of a square having sides of 20 cm. Approximate the materials that will be
wasted.
a. 400.00 sq. cm b. 314.00 sq. cm c. 86.00 sq. cm d. 22.80 sq. cm
15. A circle with radius 12 cm and a rectangle with width 16 cm have equal areas. Which of the following is the
approximation of the length of the rectangle?
a. 4.19 cm b. 9.00 cm c. 4.71 cm d. 28.26 cm
16. 2. One hectare is equivalent to 10,000 square meters. How many hectares are in a rectangular field which
is 750 m wide and 800 m long?
A. 120 B. 56 C. 60 D. 40
17. An aquarium is 40 cm high. Its length is twice its height, while its width is half its height. How much water
is needed to fill in the aquarium in cu. cm.?
A. 64,000 B. 32,000 C. 16,000 D. 8,000
18. What is the volume of a cube if one face has a perimeter of 36 cm?
A. 729 cu. cm. B. 216 cu. cm. C. 46,656 cu. cm D. 5,832 cu. cm
19. How much water can be filled with a cone whose diameter is 24 cm and whose height exceeds 2/3 of the
radius by 2 cm?
A. 480  cu. cm. B. 540  cu. cm. C. 660  cu. cm. D. 900  cu. cm.

LICENSURE EXAMINATION FOR TEACHERS (LET)

Refresher Course

Content Area: MATHEMATICS


Focus: Algebra
Prepared by: Daisy de Borja-Marcelino
LET Competencies:

 Solve for the roots of a given quadratic equation


 Solve problems on quadratic equations
 Determine an equation given a set of roots which are imaginary/complex numbers
 Perform operations involving exponential and logarithmic functions
 Solve for the solution set of a given inequality
 Determine the rth term of the expansion (a + b)n
 Solve problems involving variations
 Determine the number of positive and negative roots of a given polynomial

Equations

An equation that contains at least one variable is called an open sentence. Equations b & c above are
examples of open sentences. In equation b, only -1 makes the sentence true or satisfies the equation. However,
more than one number might satisfy an equation. For example, +2 and -2 satisfy the equation x  4  0 . Any
2

number that satisfies an equation is called a solution or root to the equation. The set of numbers from which you
can select replacements for the variable is called the replacement set. The set of all solutions to an equation is called
the solution set to the equation. To solve an equation means to find all of its solutions.

QUADRATIC EQUATION
An equation of the form ax2 + bx + c = 0 where a  0, a ,b, and c are constants, is a quadratic equation.

ROOTS OF QUADRATIC EQUATIONS


To solve a quadratic equation means to find the value of x (unknown) that will satisfy the given equation.
The values of x that will make the equation true are called the roots or solution of the quadratic equation.

Methods of Finding the Roots of a Quadratic Equation

1. Factoring (Use this method if the quadratic equation is factorable)

Example: Determine the roots of x2 -8x - 15 = 0

Solution: Factoring the left side of the equation,


(x - 5) (x – 3) = 0

Equating each factor to zero,


(x - 5) = 0 (x – 3) = 0
x=5 x=3
2. Quadratic Formula
 b  b 2  4ac
The quadratic formula is x
2a

BINOMIAL FORMULA

To obtain the terms of the binomial expansion (a + b) n, we use the binomial formula:
n 1 n (n  1)a n  2 b 2 n (n  1)( n  2)a n 3 b 3
n
(a + b) = a  na
n
b   ...  nab n 1  b n
2! 3!

THE rth TERM OF THE EXPANSION (a + b)n

n (n  1)( n  2)...( n  r  2)a n  r 1 b r 1


rth term =
(r  1)!

EXPONENTIAL FUNCTIONS

The exponential function f with base b is denoted by f(x) = bx,


where b > 0 , b  1, and x is any real number.

Properties of f(x) = bx

 f has the set of real numbers as its domain.


 f has the set of positive real numbers as its range.
 f has a graph with a y-intercept of (0,1).
 f is a one-to-one function.
 f has a graph asymptotic to the x-axis.
 f is an increasing function if b>1 and f is a decreasing function if 0<b<1.

An exponential function has a constant base and a variable exponent.

The Natural Exponential Function

For all real numbers x, the function defined by f ( x)  e is called the natural exponential function. Note
x

that e is an irrational number and its accurate value to eight places is 2.71828183.

EXPONENTIAL EQUATIONS
An equation where the unknown quantity appears in an exponent is called an exponential equation.

SOLVING EXPONENTIAL EQUATIONS


To solve an exponential equation is to find the value of the unknown quantity in the given equation.

LOGARITHMIC FUNCTIONS

For x > 0, b > 0, and b  1, we have y = log a(x) if and


only if ay = x.
Note:
1. If the base of the logarithm is not indicated it is understood that the base is 10.
2. If the base of the logarithm is the number e, then it is called a natural logarithm and it is written as f(x) = ln x.

Properties of f x   log b x

 f has the set of positive real numbers as its domain.


 f has the set of real numbers as its range.
 f has a graph with a x-intercept of (1,0).
 f is a one-to-one function.
 f has a graph asymptotic to the y-axis.
 f is an increasing function if b>1 and f is a decreasing function if 0<b<1.

Remarks

 Logarithmic functions are the inverse of exponential functions.


 We can use the rules of exponents with logarithms.
 The two most common logarithms are called common logarithms and natural logarithms. Common
logarithms have a base of 10, and natural logarithms have a base of e.
 Equation in exponential form can be rewritten in logarithmic form, and vice versa.

The exponential form of y  log b x is b y  x .

The logarithmic form of b y  x is y  log b x .

Example1: The exponential equation 72 = 49 may be written in terms of a logarithmic equation as log7 (49) = 2.

1 1
Example 2: The exponential equation 9-3 = 3
or may be written in terms of a logarithmic equation as log 9
9 729
 1 
  = -3
 729 

Basic Properties of Logarithms

Property 1 : loga (1) = 0 because a0 = 1.

Example 1: In the equation 220 = 1, the base is 22 and the exponent is 0. Remember that a logarithm is an exponent,
and the corresponding logarithmic equation is log22 (1) = 0, where the 0 is the exponent.
0
2 2
Example 2: In the equation   = 1, the base is and the exponent is 0. Remember that a logarithm is an
5 5
exponent, and the corresponding logarithmic equation is log 2 1 = 0.
5

Property 2: loga (a) = 1 because a1 = a

Example 3: In the equation 71 = 7, the base is 7, the exponent is 1, and the answer is 7. Since a logarithm is an
exponent, and the corresponding logarithmic equation is log7 7 = 1

Example 4: Use the exponential equation m1 = m to write a logarithmic equation. If the base m is greater than 0,
then logm (m) = 1.

Property 3: loga (a)x = x because ax = ax

Example 5: Since 92=92, we may write the logarithmic equation with base 9 as log 9 92 = 2.

Example 6: Since you know that 112=112, we may write the logarithmic equation with base 11 as log 11112 = 2.

INEQUALITIES
Any relation expressed using the symbols <, >, > or < is called an inequality.
An absolute inequality is an inequality which is always true. A conditional inequality is one which is true
only for certain values of the variable involved.
1. 4 > 3 is an absolute inequality
2. x > 3 is a conditional inequality

PROPERTIES OF INEQUALITIES
Let a, b, c, & d be real numbers. The following hold.
1. Trichotomy Property
a > b or a < b or a = b
2. a > b if a - b > 0
a < b if a – b < 0
3.
a. If a> 0 and b> 0, then a + b> 0 and ab>0.
b. If a < 0 and b < 0, then a+b< 0 and ab> 0

4. Transitivity

If a < b and b < c then a < c.

5. Addition Property

If a < b and c < d, then a + c < b+ d

6. Multiplication Property
If a < b and c > 0, then ac < bc
If a < b and c < 0, then ac > bc
SOLVING INEQUALITIES

To solve an inequality means to find the value of the unknown that will make the inequality true.

POLYNOMIAL FUNCTION
The function defined by the equation
f(x) = a0xn +a1xn-1 + a2xn-2 + . . .+ an-2x2 + an-1x + an

where n is a nonnegative integer and a0, a1, . . ., an are constants, a0,  0 is a polynomial function in x of
degree n. The zeros or roots of f(x) are the numbers that will make f(x) = 0.

The Number of Positive and Negative Roots of a Polynomial Function


If f(x) is a polynomial function with real coefficients, then the following are true.
 The number of positive real zeros of f(x) is either equal to the number of variations in sign in f(x), or to that number
diminished by a positive even integer.

 The number of negative real zeros of f(x) is either equal to the number of variations in sign in f(-x), or to that
number diminished by a positive even integer.
LICENSURE EXAMINATION FOR TEACHERS (LET)

Refresher Course

Majorship: MATHEMATICS
Prepared by: Daisy de Borja-Marcelino
FOCUS: Mathematical Investigation and Problem Solving
LET COMPETENCIES:
1. Cite differences between problem solving and mathematical investigations.
2. State patterns observed as conjectures.
3. Solve non-routine problems.

CONTENT UPDATE
I. Problem Solving

Problem solving is defined as a set of actions to be done to perform the task or to solve the problem. It is a process
of applying acquired knowledge to a new or unfamiliar situation.

I. Polya’s Problem-Solving Principles

1. Understand the problem

Understanding the given problem is a very important principle in order to solve it correctly.
The following questions may be helpful in the analysis of a given problem:
 Do you understand all the words used in stating the problem?
 What are you asked to find or show?
 Can you restate the problem in your own words?
 Could you work out some numerical examples that would help make the problem clear?
 Could you think of a picture or diagram that might help you understand the problem?
 Is there enough information to enable you to find a solution?
 Is there extraneous information?
 What do you really need to know to find a solution?
 Is there yet another way to state the problem?
 What does key word really mean?

2. Devise a plan

The following strategies may be of great help as you learn the art of problem solving.
● guess and check ● make a table
● make an orderly list ● use a variable
● draw a diagram ● work backward
● look for a pattern ● eliminate possibilities

3. Carry out the plan

To carry out the plan you devised earlier be careful and be patient to make it work. If it doesn’t work after
several trials, then discard it and try a new strategy.

4. Look back

Looking back is an important step in developing problem-solving skills. Once you have solved the problem
make it a habit to go over your solution and polish it.

II. Mathematical Investigation

A mathematical investigation of a problem or of an open-ended situation is a sustained exploration of


the problem or situation.

Stages in a Mathematical Investigation

1. Getting Started
- Attaining familiarity with the situation to be investigated.
- Producing instances, maybe starting from the simplest or whatever is interesting.
- Deciding on what is worth pursuing.
2. Exploring Systematically
- Systematic listing/ drawing;
- Organizing relationships in tables or graphs; and
- Looking for a pattern or relationship.

3. Making Conjecture
- Making general statements about patterns or relationships observed in the cases considered.

A conjecture is a generalization obtained inductively, which has not been validated or proven true.
4. Testing/ Verifying Conjectures
- Checking consistency of conjectures using existing cases;
- Predicting results for untried cases for which data are available.

5. Explaining/ Justifying Conjectures - Explaining why the conjectures made will work for new or all cases

6. Reorganizing
- Simplifying/ generalizing the approach
- Seeing the connection among the conjectures

7. Elaborating - Extending the investigation by considering other aspects of the investigation

8. Summarizing - Involves an account or summary, written or oral, of what has been obtained in stages 2 – 7, with
some reference on the experiences in stage 1.

MAJORSHIP: MODERN GEOMETRY

LET COMPETENCIES:

1. Give characteristics of non-Euclidean geometry which are not found in Plane Euclidean Geometry.
2. Define and illustrate concepts in linear algebra
3. Apply properties of matrices in performing matrix operations
4. Evaluate determinants
5. Perform modular clock arithmetic

MODERN GEOMETRY

Non-Euclidean geometry
Non-Euclidean Geometry is any geometry that is different from Euclidean geometry. The two most common non-
Euclidean geometries are elliptic geometry and hyperbolic geometry.
A. Hyperbolic Geometry

Hyperbolic geometry is known as saddle geometry or Lobachevskian geometry. It differs in many ways from
Euclidean geometry, often leading to quite counter-intuitive results. Some of the remarkable consequences of this
geometry's unique fifth postulate include: Moreover, in this field, more than one distinct line through a particular
point will not intersect another given line..

1. The sum of the three interior angles of a triangle is strictly less than 180°. Moreover, the angle sums of two distinct
triangles are not necessarily the same.

2. Two triangles with the same interior angles have the same area.

B. Elliptic Geometry

In Elliptic geometry there are no lines that will not intersect,

Some theorems in Elliptic Geometry


1. The angle sum of any triangle is more than 180o.

2. Given two lines perpendicular to line CG. By the parallel postulate for elliptic geometry, these two lines
meet at a point A. Then every line through A is perpendicular to line CG.

C. Projective Geometry

Projective geometry is the most general and least restrictive in the hierarchy of fundamental geometries. It is
an intrinsically non-metric geometry, whose facts are independent of any metric structure. Under the projective
transformations, the incidence structure and the cross-ratio are preserved. In particular, it formalizes one of the
central principles of perspective art: that parallel lines meet at a point called an ideal point. Consequently, the five
initial axioms in Euclidean Geometry resulted to the following axioms.

1. Any two distinct points determine one and only one line.
2. Any two distinct coplanar lines intersect in one and only one point.
3. Any line not in a given plane intersects the plane in one and only one point.
4. Any two distinct planes intersect in one and only one line.
5. Any three noncollinear points, also any line and a point not on the line, determine one and only one plane.

MATRICES AND MATRIX OPERATIONS

Definition

A matrix is defined as a rectangular array of elements. The entries, also called elements, may be real, complex or
functions. If the arrangement has m rows and n columns, then the matrix is of order m x n (read as m by n). A matrix
is enclosed by a pair of parameters such as ( ) or [ ]. It is denoted by a capital letter.

 a11 a12  a1n 


a a22  a2 n 
A   21
   aij  
 
am1 am 2  amn 

A = [ aij ]

TYPES OF MATRICES

1. The ROW MATRIX: This matrix has only one row.

Example: [1 6 4 3] This is a 1 X 4 row matrix

2. The COLUMN MATRIX: This matrix has only one column.


 10 
 2
Example:   This is 4 x 1 column matrix
 7 
 
  8
3. The RECTANGULAR MATRIX: This has two or more rows with two or more columns.

Example
  2 3 5  2
0 8 1 4 
This is a 2 X 4 matrix, because it contains two rows and four

columns

 8  11 22 0 
7 6  95 10  This is a 3 X 4 matrix

0 45 49 68

4. The SQUARE MATRIX: This is a special case of a Rectangular Matrix; here the
number of rows is equal to the number of columns.
1 3 2 4
a b c 5
 1 
f 
1 3
Example: A = d e B= 
 8 1 1 1
 g h i   
8 5 3 1

Here A and B are square matrices of order 3 and 4 respectively

5. The DIAGONAL MATRIX: This is a square matrix where all its non-diagonal elements are 0
Example:
5 0 0 0
8 0 0 0
92 0    6 0 0 
(a)  0 1 (b) 0  1 0 (c) 
  0 3 0
  0 0 8
0
 
0 0 0 10
These are diagonal matrices of order 2, 3 and 4 respectively.

6. The SCALAR MATRIX: This is a diagonal matrix where all the elements on its leading diagonal to bottom right are
of equal value.
Example:
6 0 0 0
3 0 0 0
2 0   6 0 0
(a) 0 2 (b) 0 3 0 (c) 
  0 0
  0 0 3
0 6
 
0 0 0 6
7. The IDENTITY MATRIX: This is a scalar matrix where the elements on its leading diagonal (the diagonal running
from top left to bottom right) are 1 and the rest are of value 0
Example:
1 0 0 0
0 1 0 0
 This is an identity matrix I4 of order 4.
0 0 1 0
 
0 0 0 1

PROPERTIES OF MATRIX ADDITION

Theorem Let A, B, C, and D be matrices of the same size, m x n.

1. A + B = B + A (Commutativity)

2. A + (B + C) = (A + B ) + C (Associativity)

3. There is a unique m x n matrix O such that A+ 0 = A for any m x n matrix A. The matrix O is called the m
x n additive identity or zero matrix.

4. To each m x n matrix A, there is a unique m x n matrix D such that

A+D=0

We write D as (- A), so that A + D = 0 can be written as A + (-A) = 0. The matrix (-A) is called the additive inverse or
negative of A.

PROPERTIES OF MATRIX MULTIPLICATION

Theorem Let A, B, and C be matrices of the appropriate sizes.

1. A(BC) = (AB)C (Associativity)

2. A(B + C) = AB + AC (Right Distributivity)

3. (A + B)C = AC + BC (Left Distributivity)

PROPERTIES OF MULTIPLICATION BY A SCALAR

Theorem Let A and B be matrices of the appropriate sizes, and let r and s be scalars.

1. r(sA) = (rs)A (Associativity)

2. (r + s)A = rA + sA (Distributivity I)

3. r(A + B) = rA + rB (Distributivity II)

4. A(rB) = r(AB) = (rA)B


A
Definition Let A = [ai j ] be an n x n matrix. We define the determinant of A (written det(A) or ) by

det A =
A
=
 ()a 1 j1 a 2 j 2 ...a n j n

where the summation ranges over all permutations j1 j2 …jn of the set S = {1,2,…n}.The sign is taken
as + or – according to whether the permutation j1 j2…jn is even or odd.

Second-order Determinant

 a1 b1 
a b 
If A is the square matrix of order two  2 2  then the determinant of A, denoted by

a1 b1 a1 b1
a2 b2 a2 b2
either det A or ,is defined by = a1b2 – a2b1

Example:
Compute the determinant:

PROPERTIES OF DETERMINANT

1. The determinant of a matrix and its transpose are equal, that is, det(AT ) = det(A).

2. If two rows (columns) of A are equal, then det (A) = 0


3. If a row (column) of A consists entirely of zeros, then det (A) = 0

4. The determinant of a diagonal matrix is the product of the entries on its main diagonal.

5. If matrix A = [ai j] is upper (lower) triangular, then det (A) =a11a22…ann; that is
The determinant of a triangular matrix is the product of the elements on the main diagonal.

6. If a multiple of one row of A is added to another row to produce a matrix B,


then det B = det A.

7. If two rows of A are interchanged to produce B, then det B = - det A.

8. If one row of A is multiplied by k to produce B, then det B = k  det A.

9. The determinant of a product of two matrices is the product of their determinants; That is, det(A B) = det(A) det(B)

MODULAR CLOCK ARTITHMETIC

Clock (or modular) arithmetic is arithmetic you do on a clock instead of a number line. On a 12-hour clock, there are
only twelve numbers in the whole number system. However, every number has lots of different names. For example,
the number before 1 is 0, so 12 = 0 on a 12-hour clock

In clock arithmetic, you can add, subtract, and multiply, you can divide by some numbers.

Addition and Subtraction

Addition and subtraction work the same as on number line. For example, to add 9 and 7, start at 0, count 9 along
the line, and then count 7 more. You are at 16.If you count on a 12-hour clock, you will be at 4.

To add negative numbers, use the minus (-) sign to change direction. To subtract on a clock, first find standard
(positive) names for the two numbers, count clockwise for the first one, and count counter clockwise for the second.
 In ordinary arithmetic, the additive inverse of 4 is -4. In mod 12 arithmetic, the additive inverse of 4 is 8.
In either system, the sum of a number and its additive inverse is zero.
 Rows corresponding to additive inverses are opposites of one another, save for the number 0.

Addition Mod 12 ( 12- hour clock)

You might also like